Você está na página 1de 124

L ICENCIATURA EM M ATEM ÁTICA - M ÓDULO III

G EOMETRIA A NAL ÍTICA


PAULO A LEXANDRE A RA ÚJO S OUSA
PRESIDENTE DA REPÚBLICA
Luiz Inácio Lula da Silva
MINISTRO DA EDUCAÇÃO
Fernando Haddad
GOVERNADOR DO ESTADO
Wellington Dias
REITOR DA UNIVERSIDADE FEDERAL DO PIAUÍ
Luiz de Sousa Santos Júnior
SECRETÁRIO DE EDUCAÇÃO À DISTÂNCIA DO MEC
Carlos Eduardo Bielschowsky
COORDENADORIA GERAL DA UNIVERSIDADE ABERTA DO BRASIL
Celso Costa
SECRETÁRIO DE EDUCAÇÃO DO ESTADO DO PIAUÍ
Antonio José Medeiros
COORDENADOR GERAL DO CENTRO DE EDUCAÇÃO ABERTA
À DISTÂNCIA DA UFPI
Gildásio Guedes Fernandes
SUPERINTENDENTE DE EDUCAÇÃO SUPERIOR NO ESTADO
Eliane Mendonça
DIRETOR DO CENTRO DE CIÊNCIAS DA NATUREZA
Helder Nunes da Cunha
COORDENADOR DO CURSO DE MATEMÁTICA À DISTÂNCIA
João Benı́cio de Melo Neto
COODENADORA DE MATERIAL DIDÁTICO DO CEAD/UFPI
Cleidinalva Maria Barbosa Oliveira
2
APRESENTAÇÃO

Este texto é destinado aos estudantes aprendizes que participam do


programa de Educação à Distância da Universidade Aberta do Piauı́
(UAPI), vinculada ao consórcio formado pela Universidade Federal do
Piauı́ (UFPI), Universidade Estadual do Piauı́ (UESPI) e Centro Fede-
ral de Ensino Tecnológico do Piauı́ (CEFET-PI), com apoio do Governo
do estado do Piauı́, através da Secretaria de Educação.

O texto é composto de quatro unidades, contendo itens e subitens,


que discorrem sobre: Geometria Analı́tica do Plano e do Espaço Eu-
clidiano tridimensional, Secções Cônicas e Superfı́cies Quádricas.

Na unidade 1, introduzimos um sistema de coordenadas carte-


sianas em um plano qualquer, mostrando a importância deste con-
ceito. Como consequência disto, podemos estudar geometria utilizan-
do métodos algébricos - donde surge o conceito de Geometria Analı́ti-
ca.

Apresentamos o conceito de vetor e as operações de adição de


vetor e multiplicação de vetor por escalar, com as suas principais pro-
priedades e consequências.

Finalizamos a unidade mostrando ao leitor uma aplicação do con-


teúdo trabalhado.

Na unidade 2, apresentamos a definição geométrica de cônicas


e obtemos equações analı́ticas que as representam. As equações
obtidas são equações do segundo grau a duas variáveis. Então, surge
o questionamento: qual cônica uma equação (geral) do segundo grau

2
3

a duas variáveis representa? Nesta unidade, também apresentamos


ferramentas necessárias para respondermos a esta pergunta.

Finalizamos a unidade mostrando ao leitor situações cotidianas


onde as cônicas - elipse, parábola e hipérbole - aparecem natural-
mente.

Na unidade 3, introduzimos a Geometria Analı́tica no espaço, sem-


pre fazendo uso da intuição geométrica que temos sobre o espaço
que nos rodeia, o que nos possibilita uma interpretação geométrica
dos sistemas lineares 3 × 3 - as incógnitas x, y, z são as coordenadas
de um ponto no espaço de três dimensões e cada uma das equações
representa um plano nesse espaço.

Concluı́mos a unidade mostrando, por meio de um exemplo, onde


aplicamos a teoria estudada no nosso dia a dia.

Na unidade 4, introduzimos a definição algébrica das superfı́cies


quádricas, fazendo uso de equações do segundo grau a três variáveis,
e apresentamos as ferramentas necessárias para identificarmos que
superfı́cie quádrica uma equação do segundo grau a três variáveis
representa.

Para alcançarmos nosso objetivo, ou seja, identificarmos as quádri-


cas, utilizamos o Teorema Espectral - Diagonalização de Matrizes
Simétricas.
4

SUMÁRIO GERAL

UNIDADE - 1: O plano

(1.1) Introdução

(1.2) Sistema de coordenadas

(1.3) Distância entre pontos

(1.4) Operações com vetores

(1.5) Produto interno e ângulo entre vetores

(1.6) Equação da reta

(1.7) Aplicação

(1.8) Nota histórica

(1.9) Exercı́cios resolvidos

(1.10) Exercı́cios propostos

(1.11) Referências Bibliográficas

UNIDADE - 2: Cônicas

(2.1) Introdução

(2.2) Circunferência

(2.3) Parábola

(2.4) Elipse

(2.5) Hipérbole

(2.6) Rotação de um conjunto

(2.7) Aplicação

(2.8) Nota histórica


5

(2.9) Exercı́cios resolvidos

(2.10) Exercı́cios propostos

(2.11) Referências Bibliográficas

UNIDADE - 3: O espaço

(3.1) Introdução

(3.2) Sistema de coordenadas no espaço

(3.3) Distância entre pontos no espaço

(3.4) Vetores no espaço

(3.5) Produto vetorial

(3.6) Equação do plano

(3.7) Distância de um ponto a um plano

(3.8) Retas reversas

(3.9) Aplicação

(3.10) Exercı́cios resolvidos

(3.11) Exercı́cios propostos

(3.12) Referências Bibliográficas

UNIDADE - 4: Quádricas centrais

(4.1) Introdução

(4.2) Equações padrão

(4.3) Autovalores e autovetores

(4.4) Conjunto ortonormal

(4.5) Identificação de uma quádrica central


6

(4.6) Equação geral do segundo grau

(4.7) Nota histórica

(4.8) Exercı́cios resolvidos

(4.9) Exercı́cios propostos

(4.10) Referências Bibliográficas


RESUMO UNIDADE 1

Nesta unidade, introduzimos um sistema de coordenadas carte-


sianas em um plano qualquer, mostrando a importância deste con-
ceito. Como consequência disto, podemos estudar geometria utilizando
métodos algébricos - donde surge o conceito de Geometria Analı́tica.

Apresentamos o conceito de vetor e as operações de adição de


vetor e multiplicação de vetor por escalar, com as suas principais pro-
priedades e consequências.

Finalizamos a unidade mostrando ao leitor uma aplicação do con-


teúdo trabalhado.
SUMÁRIO

(1.1) Introdução

(1.2) Sistema de coordenadas

(1.3) Distância entre pontos

(1.4) Operações com vetores

(1.5) Produto interno e ângulo entre vetores

(1.6) Equação da reta

(1.7) Aplicação

(1.8) Nota histórica

(1.9) Exercı́cios resolvidos

(1.10) Exercı́cios propostos

(1.11) Referências Bibliográficas


1. O plano

1.1 Introdução

René Descartes, matemático e filósofo, nasceu em La Have,


França, em 31 de março de 1596. Morreu em Estolcomo, em 1◦ de
fevereiro de 1650. É considerado um dos fundadores da filosofia mo-
derna. Uma de suas contribuições à matemática foi estabelecer uma
corres- pondência biunı́voca entre os pontos de um plano e pares
de números reais, dando assim origem à geometria analı́tica. Tal
matéria tem como objetivo estudar geometria por métodos algébricos.
Também, graças a essa grande idéia, é que podemos, por exemplo,
interpretar o comportamento de uma função através de seu gráfico
que é desenhado num sistema de coordenadas cartesianas. O termo
”cartesianas” nada mais é do que uma homenagem a seu criador.

Neste capı́tulo descreveremos o método desenvolvido por René


Descartes. Para isto, admitiremos que o leitor esteja familiarizado com
as propriedades do conjunto dos números reais, bem como, com os
resultados básico de geometria euclidiana plana.

1.2 Sistema de coordenadas

Há um princı́pio da geometria euclidiana plana que afirma:


“Fixada uma reta r, cada ponto de r corresponde a um único núme-
ro real e cada número real corresponde a um único ponto da reta r”.

9
10

Esta correspondência biunı́voca entre os pontos de uma reta e os


números reais é chamada de um sistema de coordenadas cartesianas
para a reta. O ponto cuja coordenada é zero é chamado de origem do
sistema.
Usualmente, a
+ + + + +
... −2 −1 0 1 2 ... reta orientada é
representada por
Figura 1.1: Reta orientada
uma reta horizontal
com a orientação
Definição 1.2.1. Sejam A e B dois pontos de uma reta, e, a e b suas no sentido da
respectivas coordenadas. Definimos a distância de A a B, denotada esquerda para a
por d(A, B), como sendo |a − b|. direita.

Outro princı́pio da geometria euclidiana plana diz que:


“Dada uma reta r e um ponto P ∈ r, sempre podemos escolher
um sistema de coordenadas para r de tal modo que a coordenada do
ponto P seja zero”.

Usaremos a existência de um sistema de coordenadas para uma


reta e introduziremos coordenadas em um plano.

Considere um plano π. Neste plano, escolhemos um ponto qual-


quer. Denotemo-lo por O. Passando em O, consideremos duas retas
perpendiculares e em cada uma delas um sistema de coordenadas
de tal modo que O seja origem em ambos. Chamaremos uma dessas
retas de eixo horizontal e a outra de eixo vertical.

Figura 1.2: Eixos orientados


11

A partir dessas duas retas e de O, estabeleceremos um sistema de


coordenadas para π. Colocaremos π em correspondência biunı́voca
com o conjunto R2 = {(x, y) : x, y ∈ R} da seguinte maneira: de cada
P ∈ π tracemos perpendiculares aos eixos horizontal e vertical. Essa
perpendiculares encontrarão os eixos em pontos cujas coordenadas
são, digamos, respectivamente, x e y. Associamos o ponto P ao par
ordenado (x, y).

y
....................... P

....................
x

Figura 1.3: Ponto no plano

Chamaremos a componente x do par (x, y) de abscissa de P , e a


componente y do par chamaremos de ordenada. Essas componentes
também serão chamadas de coordenadas de P .

A correspondência inversa de P → (x, y) é a seguinte: dado o par


(x, y), consideremos no eixo horizontal o ponto cuja coordenada é x
e no eixo vertical o ponto cuja coordenada é y. Por cada um desses
Saiba mais sobre
pontos conduzimos a perpendicular ao eixo. Essas perpendiculares
correspondência bi-
cruzar-se-ão num ponto P . Associamos o par (x, y) ao ponto P . Esta
unı́voca no sı́tio:
é a correspondência inversa.
www.ptmat.fc.ul.pt/
∼lsequeir/tmf/ A correspondência biunı́voca que acabamos de descrever chama-
teoricas/Fev18.pdf se sistema de coordenadas cartesianas para o plano π. Usaremos a
notação P = (x, y) para simbolizar que o ponto P tem coordenadas
(x, y). Comumente, o eixo horizontal também é chamado de eixo das
abscissas, eixo dos x ou eixo x, e o eixo vertical é chamado de eixo
das ordenadas, eixo dos y ou eixo y. É também comum chamar o
sistema de coordenadas acima descrito de sistema de coordenadas
12

xy e o plano de plano xy.

Dado um sistema de coordenadas xy num plano, chamaremos de


1◦ quadrante a região do plano formada pelos pontos cuja abscissa e a
ordenada são não negativas; de 2◦ quadrante a região formada pelos
pontos cuja abscissa é não positiva e a ordenada é não negativa; de 3◦
quadrante a região formada pelos pontos cuja abscissa e a ordenada
são não positivas; e, de 4◦ quadrante a região formada pelos pontos
cuja abscissa é não negativa e a ordenada é não positiva.

2° quadrante 1°quadrante

3° quadrante 4° quadrante

Figura 1.4: Quadrantes do plano

Dizemos que os quadrantes 1◦ e 2◦ são opostos, respectivamente,


Só para enfatizar, o
aos quadrantes 3◦ e 4◦ . E os conjuntos definidos como {(x, y) : x = y}
primeiro quadrante
e {(x, y) : y = −x} são as bissetrizes, respectivamente, dos quadran-
é oposto ao ter-
tes ı́mpares e pares.
ceiro e o segundo,
O emprego de coordenadas no plano serve a dois propósitos. O é oposto ao quarto
primeiro é atribuir um significado geométrico a fatos de natureza numé- quadrante.
rica; o segundo propósito é recorrer a tais coordenadas para resolver
problemas de geometria.

1.3 Distância entre pontos

Sejam (a1 , a2 ) e (b1 , b2 ), respectivamente, as coordenadas de dois


pontos A e B de um plano. Determinaremos a distância de A a B em
termos de suas coordenadas.
13

................................
B
..............................................

.........................
..............................................
A C

Figura 1.5: Distância entre pontos

Seja C o ponto de coordenadas (b1 , a2 ). Usando a definição de


distância entre pontos de uma reta temos que: d(A, C) = |a1 − b1 | e
d(B, C) = |a2 − b2 |. Além disso, o triângulo ABC é retângulo em C.
Utilizando o Teorema de Pitágoras, podemos concluir que
p
d(A, B) = (a1 − b1 )2 + (a2 − b2 )2 .

Exemplo 1.3.1. Calculemos a distância entre os pontos A = (1, 2) e


B = (−3, 5). Observe que,
p
d(A, B) = (1 − (−3))2 + (2 − 5)2
p
= (1 + 3)2 + (−3)2

= 16 + 9 = 5.

Exemplo 1.3.2. Dados os pontos A = (4, 5), B = (−2, 8) e C = (5, 7),


veja que o triângulo ABC é retângulo e seus catetos são AB e AC.
√ √
De fato, um cálculo simples mostra que d(A, B) = 45, d(A, C) = 5

Você lembra o e d(B, C) = 50. Portanto, d2 (A, B) + d2 (A, C) = d2 (B, C). Ou seja, o
enunciado do Teo- triângulo ABC satisfaz o Teorema de Pitágoras, logo é retângulo.
rema de Pitágoras?
Definição 1.3.1. O comprimento do segmento de extremidades A e B
Qual é?
é definido como a distância de A a B.

Definição 1.3.2. Chamamos de ponto médio do segmento AB o ponto


M deste segmento tal que d(A, M) = d(M, B).

Proposição 1.3.3. O ponto médio do segmento de reta cujas extremi-


dades são os pontos A = (a1 , a2 ) e B = (b1 , b2 ) é o ponto
a + b a + b 
1 1 2 2
M= , .
2 2
14

Demonstração. Um cálculo simples mostra que d(A, M) = d(M, B), o


que significa que M é o ponto médio de AB.

1.4 Operações com vetores

Dados A, B ∈ R2 , chama-se vetor com ponto inicial em A e ponto


−→
final em B, denotado por AB, o segmento orientado no sentido de A
para B. Veja sua representação geométrica:

Figura 1.6: Vetor

Dado um par ordenado (x1 , x2 ). Além de ser interpretado como as


coordenadas de um ponto do plano, ele também pode ser visto como
um vetor cujo ponto inicial é a origem do sistema de coordenadas e o
ponto final é o ponto do plano correspondente a tais coordenadas.
...........................................

.................................
y

Figura 1.7: Coordenadas de um vetor

Quando (x1 , x2 ) = (0, 0), estamos considerando um vetor “degene-


rado”, isto é, o vetor cujo ponto final coincide com o ponto inicial.

Definição 1.4.1. Sejam A = (a1 , a2 ) e B = (b1 , b2 ) vetores de R2 .


Definimos a soma de A com B como sendo

A + B = (a1 + b1 , a2 + b2 ).
15

Sabendo que cada coordenada do vetor A + B é a soma das co-


ordenadas correspondentes de A e B, é fácil deduzir as propriedades
formais da adição de vetores a partir de suas análogas para adição de
números reais. Tem-se assim, para quaisquer A, B, C ∈ R2 :

1. (Associatividade) A + (B + C) = (A + B) + C;

2. (Comutatividade) A + B = B + A;

3. (Elemento Neutro) Usando a notação O = (0, 0), obtemos que


A + O = A, para todo A ∈ R2 . Assim, o elemento O é chamado
de elemento neutro em relação à operação adição;

4. (Elemento Oposto) Dado A = (a1 , a2 ), denotaremos por −A o


par ordenado (−a1 , −a2 ), isto é, −A = (−a1 , −a2 ). Segue que,
A + (−A) = O. Chamaremos −A de elemento oposto a A.

Definição 1.4.2. Dados A, B ∈ R2 , definimos A menos B como sendo


A − B = A + (−B).

Exemplo 1.4.1. Dados A = (2, −1) e B = (−1, 1), calculemos A + B


e A − B. Utilizando a definição de adição de vetores e a propriedade
do elemento oposto temos:

Em se tratando de A + B = (2, −1) + (−1, 1) = (2 + (−1), −1 + 1) = (1, 0);

matemática, como A − B = A + (−B) = (2, −1) + (1, −1) = (3, −2).


tudo na vida, não
Definição 1.4.3. Dados A = (a1 , a2 ) ∈ R2 e λ ∈ R. Definimos a
devemos acreditar
multiplicação de A por λ pondo
em tudo que é
dito. Por isso, o λ · A = (λ · a1 , λ · a2 ).

leitor curioso logo


Resultam imediatamente, da definição acima, as seguintes pro-
tentará verificar
priedades:
a veracidade das
propriedades que 1. λ1 (λ2 A) = (λ1 λ2 )A, para quaisquer A ∈ R2 e λ1 , λ2 ∈ R;

envolvem opera- 2. (λ1 + λ2 )A = λ1 A + λ2 A, para quaisquer A ∈ R2 e λ1 , λ2 ∈ R;


ções com vetores.
3. λ(A + B) = λA + λB, para quaisquer A, B ∈ R2 e λ ∈ R;
16

4. 1 · A = A, para todo A ∈ R2 .

Como consequência das propriedades apresentadas acima, temos


o seguinte resultado: o ponto médio de um segmento cujas extremi-
dades são A e B é M = 21 (A + B). De fato, suponha que A = (a1 , a2 )
e B = (b1 , b2 ), então:
a + b a + b 
1 1 2 2
M = ,
2 2
1 1 
= (a1 + b1 ), (a2 + b2 )
2 2
1
= (a1 + b1 , a1 + b2 )
2
1
= (A + B).
2

Apresentaremos uma interpretação geométrica para a adição de


vetores e multiplicação de vetor por escalar. Para isso, necessitamos
de alguns resultados preliminares.

Teorema 1.4.4. Sejam A, B, C, D pontos de R2 . Então, o quadrilátero


ABCD é um paralelogramo se, somente se, B − A = C − D.

B C

A D

Figura 1.8: Paralelogramo

Demonstração. Lembremos que quatro pontos A, B, C, D de um plano


são vértices, nesta ordem, de um paralelogramo se, somente se, suas
diagonais BD e AC têm o mesmo ponto médio. Agora observe que,

1 1
B − A = C − D ⇔ B + D = A + C ⇔ (B + D) = (A + C).
2 2

A última igualdade significa que os segmentos BD e AC têm o


mesmo ponto médio.
17

Sejam A, B ∈ R2 vetores não nulos e não colineares, ou seja, os


pontos A, B e a origem O não são colineares. Considere o vetor A+B.
Como

A = A − O = A − [(−B) + B] = [A − (−B)] − B = (A + B) − B,

segue do teorema 1.4.4 que os pontos O, A, A + B, B são vértices,


nesta ordem, de um paralelogramo, o qual chamaremos de paralelo-
gramo determinado por A e B.

B A+B

ANOTE:
................................ O A
................................
Figura 1.9: Vetor soma
................................
................................
Desse modo, a adição de dois vetores A e B, não nulos e não
................................
colineares, é um vetor que se situa numa diagonal do paralelogramo
................................
determinado por A e B.
................................
Dados A, B, C, D ∈ R2 , dizemos que os segmentos AB e CD são
paralelos se estão contidos em retas paralelas, e colineares, se estão
−→
contidos na mesma reta. Agora considere segmentos orientados AB
−−→
e CD, diz-se que eles têm a mesma orientação quando são paralelos
ou colineares e os segmentos AC e BD não se intersectam. Esta
definição não faz sentido quando os segmentos AB e CD são coli-
neares. Neste caso, diz-se que eles têm mesma orientação se uma
⇀ ⇀
das semi-retas AB e CD está contida na outra.

A B

C D

Figura 1.10: Mesma orientação


18

−→ −−→
Definição 1.4.5. Diremos que dois vetores AB e CD são equipolentes
−→ −−→
e escrevemos AB ∼ CD, se têm mesmo comprimento, são paralelos
ou colineares, e têm mesma orientação.
−→ −−→ −→ −−→
Proposição 1.4.6. Sejam AB e CD vetores em R2 . Então, AB ∼ CD
se, somente se, B − A = D − C.

A demonstração desta proposição, no caso que AB e CD não são


colineares, segue como consequência do teorema 1.4.4. Deixaremos
a demonstração do outro caso, que os AB e CD são colineares, a
cargo do leitor.

Definição 1.4.7. Dados A = (a1 , a2 ) e B = (b1 , b2 ), diremos que o


−→
par ordenado (b1 − a1 , b2 − a2 ) são as coordenadas do vetor AB e
−→
escrevemos AB = (b1 − a1 , b2 − a2 ). Pela proposição 1.4.6 temos que
as coordenadas de dois vetores equipolentes são iguais.
−→
Corolário 1.4.1. Dados um vetor AB e um ponto P em R2 , existe um
−→ −→
único ponto Q tal que AB ∼ P Q.

Demonstração. Observe que B − A = (B − A) + (P − P ) = Q − P ,


onde Q = (B − A) + P . Pela proposição 1.4.6 segue o resultado.
−→
Como vimos no corolário acima, dados o vetor AB e o ponto P ,
−→ −→
existe um único ponto Q tal que AB ∼ P Q, onde Q = (B − A) + P .
−→
Diz-se que o vetor AB transportou o ponto P até a posição Q. Aliás,
a palavra vetor provém do latim vehere, que significa transportar.

Q=(B−A)+P B

P A

Figura 1.11: Transporte de ponto


19

Proposição 1.4.8. Três pontos A = (a1 , a2 ), B = (b1 , b2 ) e C = (c1 , c2 )


estão alinhados (são colineares) se, somente se,

a1 a2 1

Um boa atividade
b b 1 = 0.

para o aprendizado
1 2

c1 c2 1

do leitor é tentar
provar a proposição
nos casos não
Demonstração. Vamos supor que a1 < b1 < c1 e a2 < b2 < c2 , os
contemplados na
outros casos são tratados de modo análogo. Os pontos A = (a1 , a2 ),
demonstração.
B = (b1 , b2 ) e C = (c1 , c2 ) são colineares se, somente se, os triângulos
∆ACE e ∆ABD são semelhantes, onde D = (b1 , a2 ) e E = (c1 , a2 ).
Como os triângulos ∆ACE e ∆ABD são retângulos em E e D, res-
pectivamente, temos que os mesmos são semelhantes se, somente
AE CE
se, AD
= BD
.
........................................................

.............................................. C
............................................

................................................
B

...................................................
.........................

A D E

Figura 1.12: Pontos colineares

Observemos agora que,


AE CE c1 − a1 c2 − a2
= ⇔ =
AD BD b1 − a1 b2 − a2
⇔ (c1 − a1 )(b2 − a2 ) = (b1 − a1 )(c2 − a2 )

⇔ a1 (b2 − c2 ) − b1 (a2 − c2 ) + c1 (a2 − b2 ) = 0



b2 1 c2 1 b2 1

⇔ a1 − b1 + c1 =0
c2 1 a2 1 a2 1


a1 a2 1


⇔ b1 b2 1 = 0.


c1 c2 1

20

−→ −−→
Sejam AB e CD dois vetores paralelos ou colineares, então os
−−→ −−→
vetores OP1 e OP2 , onde P1 = B − A e P2 = D − C, são colineares.
Donde concluı́mos que os pontos O, P1 e P2 estão alinhados. Pela
proposição acima, este fato ocorre somente quando existe λ ∈ R tal
que B − A = λ(D − C).
−→ −−→
Consequência: Dois vetores AB e CD são paralelos ou colineares
se, somente se, existe λ ∈ R tal que B − A = λ(D − C).
Considere A = (a, b), não nulo, e λ ∈ R. Segue da proposição 1.4.8
que os vetores A e λA são colineares, isto é, os pontos O = (0, 0),
A = (a, b) e λA = (λa, λb) são colineares.

Quanto à orientação, se λ > 0 então λa e a têm mesmo sinal,


assim como λb e b. Isto diz que os vetores A e λA estão no mesmo
quadrande, e por conseguinte, têm mesmo sentido. Se λ < 0, λa e
a têm sinais contrários, assim como λb e b. Consequentemente, os
vetores A e λA estão em quadrandes opostos, e por conseguinte, têm
sentidos contrários.

y .A
A

Figura 1.13: Multiplicação por escalar


A distância de A à origem é igual a d(O, A) = a2 + b2 . Por con-
seguinte, a distância de λA = (λa, λb) à origem é igual a :

p √
d(O, λA) = (λa)2 + (λb)2 = |λ| a2 + b2 = |λ|d(O, A).

A análise acima diz que λA é um vetor que tem a mesma direção


de A e cujo comprimento é igual a |λ| vezes o comprimento de A.
21

Quanto à orientação, terá a mesma se λ > 0 e terá sentido contrário


se λ < 0.

1.5 Produto interno e ângulo entre vetores

A fórmula da distância entre dois pontos, dada em termos das


coordenadas desses pontos, serve de partida para um grande número
de resultados da Geometria Analı́tica. Vejamos um exemplo.

Dados dois pontos P = (a1 , a2 ) e Q = (b1 , b2 ) obteremos a condição,


em termos dessas coordenadas, que assegura o perpendicularismo
dos segmentos OP e OQ.

Pelo Teorema de Pitágoras, os segmentos OP e OQ são perpen-


diculares se, somente se , d2 (P, Q) = d2 (O, P ) + d2 (O, Q).

P
Q

ANOTE:
O
................................
................................
Figura 1.14: Segmentos perpendiculares
................................
................................
A fórmula da distância entre dois pontos nos permite escrever esta
................................
equação como
................................
................................ (b1 − a1 )2 + (b2 − a2 )2 = a21 + a22 + b21 + b22 .

Simplificando a expressão encontrada temos: a1 b1 + a2 b2 = 0. Esta é


condição de perpendicularismo entre dois vetores.

Conclusão: Dois vetores P = (a1 , a2 ) e Q = (b1 , b2 ) são perpendicu-


lares (notação P ⊥ Q) se, somente se, a1 b1 + a2 b2 = 0.

De uma forma mais geral, dados os pontos A = (a, b), A′ = (a′ , b′ ),


C = (c, d) e C ′ = (c′ , d′ ), com A 6= A′ e C 6= C ′ , temos que os segmen-
22

tos AA′ e CC ′ são perpendiculares se, somente se, os segmentos OP


e OQ também o são, onde P = (a′ − a, b′ − b) e Q = (c′ − c, d′ − d).
Assim, a condição de perpendicularismno dos segmentos AA′ e CC ′ ,
se exprime, em termos das coordenadas desses segmentos, como

(a′ − a)(c′ − c) + (b′ − b)(d′ − d) = 0.

C’

A’

A
Q P

Figura 1.15: Segmentos sem extremidade comum

Exemplo 1.5.1. Voltemos ao exemplo 1.3.2. Se A = (4, 5), B = (−2, 8)


e C = (5, 7) então, os segmentos AB e AC são perpendiculares. De
fato,
(−2 − 4)(5 − 4) + (8 − 5)(7 − 5) = −6 · 1 + 3 · 2 = 0.

Definição 1.5.1. Sejam A = (a1 , a2 ) e B = (b1 , b2 ). Definimos o pro-


duto interno de A por B como sendo:

hA, Bi = a1 b1 + a2 b2 .

Deste modo, dois vetores A e B em R2 são perpendiculares se,


somente se, hA, Bi = 0.

Exemplo 1.5.2. Dado o vetor (a, b), temos que o vetor (−b, a) é sem-
pre perpendicular a ele, ou seja, h(a, b), (−b, a)i = a(−b) + ba = 0.

As propriedades apresentadas abaixo resultam imediatamente da


definição 1.5.1.
23

1. hu, vi = hv, ui, para quaisquer u, v ∈ R2 ;

2. hu, v + wi = hu, vi + hu, wi, para quaisquer u, v, w ∈ R2 ;

3. hλu, vi = λhu, vi para quaisquer u, v ∈ R2 e λ ∈ R;

4. hu, ui ≥ 0, para todo u ∈ R2 .

Definição 1.5.2. A norma de um vetor A = (a1 , a2 ) é definida como


p q
|A| = hA, Ai = a21 + a22 .

Lema 1.5.1. Dados A ∈ R2 e λ ∈ R, temos que:

1. |A| ≥ 0;

2. |λA| = |λ| · |A|.

Demonstração. A prova do item (1) segue do fato que hA, Ai ≥ 0.


p p
Agora veja que, |λA| = hλA, λAi = λ2 hA, Ai = |λ| · |A|.

Dado um vetor A ∈ R2 , não nulo, o item (2) do lema acima nos


fornece uma maneira de encontrarmos um vetor B com a mesma
direção do vetor A e norma igual a |α|, para qualquer α ∈ R. De
α
fato, escolhendo λ = |A|
temos que vetor B = λA satisfaz

|α|
|B| = |λ| · |A| = |A| = |α|.
|A|
Exemplo 1.5.3. Acharemos um vetor com norma 2, mesma direção e
2
sentido contrário ao do vetor A = (1, 2). Tomando λ = − |A| = − √25 , o
Augustin - Louis sinal negativo é devido à exigência do sentido ser contrário, então o
 
Cauchy (1789- vetor B = λA = − √2 , − √4 é o vetor procurado.
5 5
1857) deixou cerca
Já o item (1), do lema anterior, será usado para provarmos uma
de 800 trabalhos
ferramenta (teorema abaixo) muito útil na resolução de problemas
entre livros e arti-
matemáticos.
gos, cobrindo quase
todos os ramos da Teorema 1.5.3 (Desigualdade de Cauchy-Schwarz). Para quaisquer
2
matemática, um vetores A, B ∈ R vale a seguinte desigualdade
feito talvez só
|hA, Bi| ≤ |A| · |B|.
superado por Euler.
24

Demonstração. Pelo item (1) do lema 1.5.1, temos que

hA + tB, A + tBi ≥ 0, ∀ t ∈ R.

Segue que p(t) = |B|2 · t2 + 2hA, Bit + |A|2 ≥ 0, para todo t ∈ R. Então,
o discriminante da função quadrática p é não positivo. Portanto,

4hA, Bi2 − 4|A|2 |B|2 ≤ 0 ⇒ |hA, Bi| ≤ |A| · |B|.

Corolário 1.5.1 (Desigualdade Triangular). |A + B| ≤ |A| + |B|, para


quaisquer dois vetores A, B ∈ R2 .

Demonstração. Primeiramente observe que |A + B| ≤ |A| + |B| ⇔


|A + B|2 ≤ (|A| + |B|)2 . Então basta provar a segunda desigualdade.
Assim,

|A + B|2 = hA + B, A + Bi

= hA, Ai + 2hA, Bi + hB, Bi

= |A|2 + 2hA, Bi + |B|2

≤ |A|2 + 2|A| · |B| + |B|2

= (|A| + |B|)2 .

A condição de perpendicularismo é um caso particular da fórmula


que dá o cosseno do ângulo entre dois segmentos. Pelo que foi visto
anteriormente, dois vetores são perpendiculares se, somente se, o
produto interno deles é igual a zero. Levados por esta observação,
vamos obter a fórmula do cosseno do ângulo entre dois segmentos.

Sejam P = (a, b) e Q = (x, y) pontos distintos e α e β as medidas


em radianos dos ângulos do eixo x com os segmentos OP e OQ,
respectivamente. Então, os segmentos OP ′ e OQ′ também formam
ângulos α e β, respectivamente, com o eixo x, onde P ′ e Q′ são dados
abaixo:
a b x y
P ′ = (√ ,√ ); Q′ = ( p ,p ).
a2+b2 2
a +b2 x2 + y 2 x2 + y 2
25

1 Q’
P

0 P’

Figura 1.16: Ângulo entre segmentos

Caro leitor, é de fácil Observe que o pontos P ′ e Q′ estão situados à distância 1 da


verificação (acon- origem O. Então, cos α = √ a , sin α = √ b , cos β = √ x
e
a2 +b2 a2 +b2 x2 +y 2
y
selhamos que o sin β = √ . Supondo β > α, temos que o ângulo do segmento OP ′
x2 +y 2
faça) que d(O, P ′) = com o segmento OQ′ mede θ = β−α. Como se sabe da trigonometria,
d(O, Q′) = 1. Basta tem-se
utilizar a fórmula
encontrada que cos θ = cos(β − α) = cos β cos α + sin β sin α.
fornece a distância Portanto,
entre dois pontos xa + yb hP, Qi
cos θ = p √ = .
x2 + y 2 · a2 + b2 |P | · |Q|
em termos de suas
Se tivermos dois segmentos de reta AA′ e CC ′ , com extremidades
coordenadas.
distintas, e quisermos obter o cosseno do ângulo entre eles em função
das coordenadas A = (a, b), A′ = (a′ , b′ ), C = (c, d) e C ′ = (c′ , d′ ),
transladaremos esses segmentos de modo a fazer A e C caı́rem sobre
a origem O, obtendo assim os segmentos OA′′ e OC ′′ , onde A′′ =
A′ − A = (a′ − a, b′ − b) e C ′′ = C ′ − C = (c′ − c, d′ − d). O ângulo entre
AA′ e CC ′ será o mesmo que entre OA′′ e OC ′′. Portanto,
−−→ −−→
(a′ − a)(c′ − c) + (b′ − b)(d′ − d) hAA′ , CC ′ i
cos θ = p p = −−→ −−→ .
(a′ − a)2 + (b′ − b)2 · (c′ − c)2 + (d′ − d)2 |AA′ | · |CC ′ |

Deve-se observar que os segmentos AA′ e CC ′ têm extremidades


distintas, o ângulo entre eles só fica bem definido quando os orienta-
mos, isto é, quando especificamos em cada um deles qual é o ponto
inicial e o ponto final. No argumento acima, a discussão admitiu que
26

os pontos iniciais dos segmentos AA′ e CC ′ são A e C. Caso A′ fosse


o ponto inicial do primeiro segmento e C o ponto inicial do segundo,
o ângulo entre eles seria o suplemento de θ e o cosseno mudaria de
sinal.
√ √
Exemplo 1.5.4. Dados os pontos A = (1, 2), B = (1 + 3, 2 + 3) e
√ √ −→ √ √ −→ √ √
C = (2+ 3, 3− 3). Veja que AB = ( 3, 3) e AC = (1+ 3, 1− 3),
então o cosseno do ângulo formado pelos segmentos AB e AC é
√ √ √ √ √
3(1 + 3) + 3(1 − 3) 2 3 1
cos θ = q √ √ q √ √ = √ = .
2 2 2 2 4 3 2
( 3) + ( 3) (1 + 3) + (1 − 3)

Consequentemente, os segmentos AB e AC formam um ângulo


de 60◦ .

1.6 Equação da reta

Sejam P um ponto e A um vetor não nulo. Seja r a reta que passa


por P e está na direção de A, ou seja, é paralela ao vetor A.

Figura 1.17: Ponto da reta r

Seja X um ponto em R2 . Então X ∈ r se, somente se, o segmento


P X é paralelo ao vetor A. Isto implica que X ∈ r se, somente se,
existe t ∈ R tal que X − P = tA ⇔ existe t ∈ R tal que X = P + tA.

Assim sendo, o conjunto dos pontos X(t) tais que X(t) = P + tA,
em que t ∈ R, é a reta que passa no ponto P na direção do vetor A.
Chamaremos t de parâmetro e X(t) = P +tA de equação paramétrica
da reta.
27

Dados dois pontos A e B, consideremos a reta que passa por estes


Zero Absoluto: o
−→
pontos. Então esta reta tem a direção do vetor v = AB, logo, sua
kelvin (sı́mbolo:
◦ equação paramétrica é X(t) = A + tv. Observe que X(0) = A e
K) é o nome
X(1) = B. Por isto, dizemos que X(t) = A + tv, quando 0 ≤ t ≤ 1, é a
da unidade de
equação paramétrica do segmento orientado AB, começando em A e
base do Sistema
terminando em B.
Internacional de
Unidades (SI) para Exemplo 1.6.1. A reta que passa pelos pontos A = (0, 1) e B = (2, 3)
a grandeza tempe- tem a mesma direção do vetor v = −→
AB = B−A = (2, 3)−(0, 1) = (2, 2).
ratura. É sabido que Donde, sua equação paramétrica é X(t) = (0, 1) + t(2, 2) = (2t, 1 + 2t).
a dependência fun-
Mais geralmente, suponha que X = (x, y), P = (p1 , p2 ) e A = (a, b).
cional entre graus
Então, a equação paramétrica da reta r que passa por P na direção A
Celsius e graus
é (x, y) = (p1 , p2 ) + t(a, b), o que equivale a:
Kelvin é linear, que

0◦ C corresponde  x = p + at
1

.
a 273, 15 K e que  y = p + bt
2

100◦C corresponde
Estas são chamadas de equações paramétricas da reta que passa
a 373, 15◦K.
por P na direção A. Podemos, neste caso, a partir das equações
Chamamos de zero
paramétricas, eliminar t e obter uma relação envolvendo x, y, p1, p2 , a
absoluto a tem-
e b. Vejamos, multiplicando x = p1 +at por −b, multiplicando y = p2 +bt
peratura de 0◦ K.
por a e somando as duas equações obtidas temos
Qual temperatura,
em graus Celsius, −b(x − p1 ) + a(y − p2 ) = 0.
corresponde ao
Sendo (−b, a) perpendicular a (a, b), e sabendo que (a, b) está
zero absoluto?
direção de r, segue que o vetor (−b, a) também é perpendicular à
Encontre a equação
reta r. Portanto, a equação obtida acima representa a reta que passa
da reta que dá a
por P = (p1 , p2 ) e é perpendicular ao vetor A = (−b, a).
correspondência
linear entre as duas Mostraremos a seguir que a equação ax + by = c, onde a2 + b2 6= 0,
grandezas e, com representa uma reta que é perpendicular ao vetor (a, b). Com efeito,
isto, responda à suponhamos que a 6= 0, fazendo y = t obtemos ax + bt = c. Donde,
pergunta. 
 x = c − bt
a a
.
 y = 0+1·t
28

Estas são as equações paramétricas da reta r que passa no ponto


( ac , 0) e tem a direção do vetor (− ab , 1). Sendo (− ab , 1) perpendicular a
(a, b), decorre que (a, b) é perpendicular a r.

A equação ax + by = c, onde a2 + b2 6= 0, será chamada simples-


mente de equação de r e (a, b) será chamado de vetor normal a r.

Exemplo 1.6.2. Seja r a reta paralela ao eixo x e que passa pelo


ponto P = (0, c). Sendo r paralela ao eixo x temos que o vetor (0, 1) é
perpendicular a r, então sua equação é dada por:

0 · (x − 0) + 1 · (y − c) = 0 ⇒ y = c.

Dada uma reta r e P 6∈ r, sabemos que existe uma única reta s


passando por P que é perpendicular a r. Seja projr (P ) o ponto de
intersecção das retas r e s. Chamamos o ponto projr (P ) de projeção
ortogonal do ponto P sobre a reta r.

Definição 1.6.1. Dados uma reta r e P 6∈ r, definimos a distância de


P à reta r, denotada por d(P, r), como

d(P, r) = d(P, projr (P )).

Sejam ax + by = c a equação da reta r e P = (x0 , y0) um ponto


fora de r. Como o vetor (a, b) é perpendicular a r, segue que s tem a
mesma direção do vetor (a, b). Assim, a equação paramétrica de s é
X(t) = (x0 +ta, y0 +tb). Seja t tal que X(t) ∈ s, isto é, X(t) = projr (P ).
Então, a(x0 + ta) + b(y0 + tb) = c. Desta igualdade decorre que

c − ax0 − by0
t= .
a2 + b2

Portanto, d(P, r) = d(P, projr (P )) = d(P, X(t)). Consequente-


mente,
q
d(P, r) = (ta)2 + (tb)2

= |t| · a2 + b2
|ax0 + by0 − c|
= √ .
a2 + b2
29

Em suma, a distância do ponto P = (x0 , y0 ) à reta r de equação


ax + by = c é :
|ax0 + by0 − c|
d(P, r) = √ .
a2 + b2

1.7 Aplicação

Ilha do tesouro

Vamos usar vetores para resolver um problema interessante. Re-


centemente foi descoberto um manuscrito do pirata Barba Negra, des-
crevendo a localização de um tesouro enterrado por ele em uma ilha
do Caribe. O manuscrito identifica perfeitamente a ilha e dá as seguin-
tes instruções:
”...qualquer um que desembarque nesta ilha verá imediatamente
dois grandes carvalhos, que chamarei de A e B, e também uma palmei-
ra, que chamarei de C.
Caminhe de C para A contando seus passos. Chegando em A, vire
para a esquerda e dê exatamente o mesmo número de passos para
chegar ao ponto M. Volte ao ponto C. Caminhe de C para B contando
seus passos. Chegando em B, vire para a direita e dê exatamente o
mesmo número de passos para chegar ao ponto N. O ponto X está
na reta que liga M e N, e à mesma distância desses dois pontos”.

Figura 1.18: Mapa do tesouro


30

Com essas precisas informações, os exploradores chegaram á


referida ilha, mas tiveram uma desagradável surpresa. Os carvalhos
estavam lá, mas a palmeira tinha desaparecido. O tesouro parecia
perdido. Entretanto, fazia parte da comitiva um matemático que, após
breves cálculos, conseguiu descobrir o tesouro. Como ele fez?

O matemático estabeleceu na ilha, que era plana, um sistema de


coordenadas com origem em A e com o ponto B no eixo x. Ele mediu
a distância de A até B e encontrou 40 metros. Assim, ficou estabele-
cido que A = (0, 0) e B = (40, 0). Para a palmeira desaparecida ele
pôs c = (x, y).

Figura 1.19: Usando vetores para achar o tesouro

−→ −−→
Observe que os vetores AC = (x, y) e AM são perpendiculares,
−−→ −−→
então M = (y, −x). Do mesmo modo, sendo BC = (x − 40, y) e BN
−−→ −−→
perpendiculares temos que BN = (−y, x−40). Donde, N = B + BN =
(40 − y, x − 40).
31

Sendo X o ponto médio de MN , suas coordenadas são dadas por


M +N  y + 40 − y −x + x − 40 
X= = , = (20, −20).
2 2 2
Portanto, para encontrar o tesouro, bastava andar 20 metros na
direção de A para B e depois virar à direita e andar mais 20 metros.

1.8 Nota histórica

Fermat e a Geometria Analı́tica

Grandezas variáveis como velocidade, aceleração e densidade, por


exemplo, envolvendo a idéia intuitiva de intensidade, eram chamadas,
no século XIV, de ”formas”.
Nicole de Oresme (1313-1382), considerado o mais importante
matemático de sua época, talvez inspirando-se na tradição grega de
associar o contı́nuo à geometria, teve a idéia de representar grafica-
mente a variação de uma forma.
Assim, no caso de um corpo que se move a partir do repouso com
aceleração constante, marcou sobre uma reta horizontal os valores
do tempo (longitudes) e representou as velocidades correspondentes
por segmentos perpendiculares á reta (latitudes). Comprovou então
que os segmentos formam um triângulo retângulo, posto que suas ex-
tremidades superiores estão alinhadas; e que a velocidade no instante
médio é a metade da velocidade no instante final.
Segundo tudo indica, parece ter sido essa a forma em que foi usa-
da pela primeira vez a idéia de representação gráfica de uma função
mediante coordenadas. Mas tendo parado praticamente por aı́ nesse
assunto, Oresme deve ser visto apenas como um precursor da ge-
ometria analı́tica. Aliás, a criação deste novo campo dependia de
progressos matemáticos (especialmente na álgebra) que ainda de-
morariam cerca de dois séculos. Dependia ainda da genialidade de
alguém: no caso, de Pierre de Fermat (1601-1665) e René Descartes
(1596-1650), cada um a seu modo, em trabalhos independentes.
32

Francês da cidadezinha de Beaumont-de-Lomagne, Fermat cur-


sou Direito em Toulouse, em cujo parlamento começou a trabalhar
em 1631 - primeiro como advogado, posteriormente como conselhei-
ro. Pelo zelo com que se dedicava às suas atividades profissionais,
dificilmente se poderia advinhar que sua verdadeira vocação era a
matemática (cultivada, com grande talento, nas horas de lazer).

Ninguém como Fermat contribuiu tanto para o progresso da mate-


mática em sua época. Participou com grande brilho da criação da
geometria analı́tica, do cálculo diferencial e integral e da teoria das
probabilidades; e foi, sem sombra de dúvida, o grande nome da fase
inicial da moderna teoria dos números. Mas, parte por sua condição
de amador, parte por sua grande modéstia, recusava-se sistematica-
mente a publicar seus trabalhos. E se estes são conhecidos hoje, é
porque ficaram registros em margens de livros, folhas avulsas e car-
tas.
A geometria analı́tica de Fermat talvez seja um subproduto da
tarefa que empreendeu a partir de 1629 de reconstruir o desapare-
cido Lugares planos, de Apolônio, mediante referências contidas na
Coleção matemática, de Papus. E é o assunto do pequeno tratado
Introdução aos lugares planos e sólidos , concluı́do no máximo no ano
de 1636, mas só publicado em 1679.

Hygino H. Domingues

Descartes, o primeiro filósofo moderno,


e a geometria analı́tica

Ao iniciar-se o século XVII, a geometria ainda representava o grosso


da matemática. E na geometria, a contribuição de Euclides, que não
ultrapassava as figuras envolvendo a reta e o cı́rculo, predominava
soberanamente. Além do mais, a geometria grega, carecendo de
métodos gerais, “só exercitava o entendimento ao custo de fatigar
33

enormemente a imaginação”, conforme palavras de Descartes.

A época porém era de profundas transformações cientı́ficas e tec-


nológicas, razão pela qual impunha-se uma matemática mais integrada
e operacional. O primeiro grande passo nesse sentido foi a associação
da álgebra com a geometria, empreendida independentemente por
Fermat e Descartes, embrião da atual geometria analı́tica.

René Descartes (1596-1650) nasceu em La Have, pequena cidade


a sudoeste e a cerca de 300km de Paris, provı́ncia de Touraine. Seu
pai, membro da pequena nobreza da França, decidiu desde logo inves-
tir em sua educação: matriculou-o, aos 8 anos de idade, no colégio
jesuı́ta da época. Descartes, porém, sempre teve saúde extrema-
mente fágil, razão pela qual não lhe cobrava no colégio a regulari-
dade da frequência às aulas; foi nessa época que adquiriu o hábito
de permanecer na cama de manhã depois de acordado, para leituras
e meditações. Ao concluir seu curso em La Flèche, Descartes já se
perguntava: há algum ramo do conhecimento que realmente ofereça
segurança? E não vislumbrava como resposta senão a matemática,
com certeza oferecida pelas suas demonstrações. Desde muito jovem,
as preocupações de ordem filosófica se manisfestavam nele.

Aos 20 anos de idade, já graduado em Direito pela Universidade de


Poitiers, Descartes estabelece-se em Paris a fim de iniciar-se na vida
mundana, como convinha a alguém da sua posição. Mas reencontra-
se com Mersenne, que conhecera em La Flèche, e ei-lo em plena
metrópole dedicando-se á matemática com todas as suas forças por
um ou dois anos. Após seguir carreira militar, em 1629 fixa-se na
Holanda - um paı́s em que havia liberdade de pensamento - onde
vivera os vinte anos seguintes. Nesse perı́odo veio à luz sua geometia.

A obra-prima de Descartes é o Discurso do método, publicado em


1637, na qual expõe a essência de sua filosofia que, em suma, é
uma defesa do método matemático como modelo para a aquisição
do conhecimento. Essa obra inclui três apêndices, sendo um deles
A geometria.
34

Já no inı́cio de seu trabalho, introduz a notação algébrica, hoje


universalmente adotada: x, y, z, ... para as variáveis e a, b, c, ... para
as constantes. Descartes pensava nas letras como segmentos de
retas. Mas rompeu com a tradição grega ao admitir que x2 e x3 , por
exemplo, podiam ser interpretados também como segmentos de reta
e não necessariamente como uma área e um volume. Com isso foi-lhe
possı́vel mostrar que as cinco operações aritméticas correspondem a
construções elementares com régua e compasso.

O Discurso do método fez de Descartes um homem famoso ainda


em vida. O fato de ter escrito essa obra em francês (ao invés de latim,
lı́ngua cientı́fica da época) tornou mais fácil a difusão de suas idéias
filosóficas.

Hygino H. Domingues

1.9 Exercı́cios resolvidos

Q 1.1. Os pontos A = (1, 2), B = (7, 4) e C são vértices de um


triângulo ABC, retângulo em A. O vértice C pertence ao eixo das
ordenadas. Determine as coordenadas de C.

Solução: Como o ponto C pertence ao eixo das ordenadas ele é da


forma C = (0, y). Sendo ABC retângulo em A, temos

d2 (B, C) = d2 (A, B) + d2 (A, C).

Substituindo as coordenadas de cada ponto na igualdade acima temos


p p
[ (7 − 0)2 + (4 − y)2 ]2 = [ (7 − 1)2 + (4 − 2)2 ]2
p
+ [ (1 − 0)2 + (y − 2)2 ]2 .

Efetuando os cálculos concluı́mos que y = 5.

Q 1.2. Determine as coordenadas do baricentro de um triângulo ABC,


sendo A = (a1 , a2 ), B = (b1 , b2 ) e A = (c1 , c2 ).
35

Solução: Lembremos que o baricentro é o ponto de encontro das


2
medianas e que o mesmo está situado a 3
da medida de cada uma
delas, a partir do respectivo vértice. Sendo G o baricentro e M1 e
M2 os ponto médios dos lados BC e AC, respectivamente, temos que

AG = 32 AM1 e BG = 23 BM2 . Sobre a semi-reta AM1 marque um ponto
P , fora do triângulo ABC, de modo que GM1 = M1 P .

M1 P
M2 |
|
G
B
A

Figura 1.20: Baricentro G

Então, M1 = 21 (P + G) e G = 21 (A + P ). Donde obtemos a seguinte


igualdade, 23 G = M1 + 21 A. Utilizando que M1 = 12 (B +C), encontramos
G = 31 (A+B +C). Substituindo as coordenadas dos vértices podemos
concluir que
a + b + c a + b + c 
1 1 1 2 2 2
G= , .
3 3
Q 1.3. Dados os pontos A = (8, 11), B = (−4, −5) e C = (−6, 9),
obtenha o circuncentro do triângulo ABC. Lembrete: circuncentro é o
centro do cı́rculo circunscrito ao triângulo.

Solução: O circuncentro é um ponto P = (x, y) equidistante dos


vértices do triângulo. Então, d(A, P ) = d(B, P ) e d(B, P ) = d(C, P ).
Substituindo as coordenadas de cada ponto temos
 
 d(A, P ) = d(B, P )  3x + 4y = 18
⇒ .
 d(B, P ) = d(C, P )  x − 7y = −19

Resolvendo o sistema acima encontramos P = (2, 3).

Q 1.4. Prove que os pontos médios dos lados de um quadrilátero de


vértices A = (a1 , a2 ), B = (b1 , b2 ), C = (c1 , c2 ) e D = (d1 , d2 ) são
vértices, nesta ordem, de um paralelogramo.
36

Solução: Sejam M1 , M2 , M3 e M4 os pontos médios dos lados AB,


BC, CD e DA, respectivamente. Então,
a + b a + b  b + c b + c 
1 1 2 2 1 1 2 2
M1 = , , M2 = ,
2 2 2 2
c + d c + d  d + a d + a 
1 1 2 2 1 1 2 2
M3 = , , M4 = , .
2 2 2 2
Provemos que as diagonais do quadrilátero M1 M2 M3 M4 se cortam ao
meio, isto é, os seus pontos médios são coincidentes. De fato,

1 a + b + c + d a + b + c + d  1
1 1 1 1 2 2 2 2
(M1 + M3 ) = , = (M2 + M4 ).
2 4 4 2

Q 1.5. Para quais valores de a os pontos A = (2, 1), B = (a + 1, 2) e


C = (−3, −1) são vértices de um triângulo ?

Solução: Para que os pontos A, B e C sejam vértices de um triângulo


eles não devem ser alinhados, ou seja,

2 1 1



a+1 2 1 6= 0.


−3 −1 1

Resolvendo o determinante encontramos que a 6= 27 .

Q 1.6. Suponha que o vetor u = (a, b) satisfaz a igualdade hu, vi = 0,


para todo vetor v = (x, y) ∈ R2 . Mostre que a = b = 0.

Solução: Como a igualdade vale para todo vetor v = (x, y) ∈ R2 ,


podemos ecolher v = (x, y) = (a, b). Então, h(a, b), (a, b)i = a2 + b2 = 0.
Portanto, a = b = 0.

Q 1.7. Sejam A = (a1 , a2 ), B = (b1 , b2 ) e C = (c1 , c2 ) pontos não


colineares. Mostre que a área do triângulo ABC vale 12 |∆|, onde

1 1 1


∆ = a1 b1 c1 .


a2 b2 c2

37

Solução: Consideremos o vetor diretor da reta r que passa pelos


−−→
pontos B e C: BC = (c1 −b1 , c2 −b2 ). Então, o vetor v = (−c2 +b2 , c1 −b1 )
é perpendicular à r. Assim, a equação de r é

(−c2 + b2 )(x − b1 ) + (c1 − b1 )(y − b2 ) = 0.

Simplificando: (−c2 + b2 )x + (c1 − b1 )y = b2 c1 − b1 c2 . A medida da


altura de ABC relativa ao lado BC é igual à distância de A à reta r.
Calculemos d(A, r).
|a1 (−c2 + b2 ) + a2 (c1 − b1 ) − (b2 c1 − b1 c2 )|
d(A, r) = p
(−c2 + b2 )2 + (c1 − b1 )2
|(b1 c2 − b2 c1 ) − (a1 c2 − a2 c1 ) + (a1 b2 − a2 b1 )|
= p
(c2 − b2 )2 + (c1 − b1 )2
|(b1 c2 − b2 c1 ) − (a1 c2 − a2 c1 ) + (a1 b2 − a2 b1 )|
= .
d(B, C)
Como a área de ABC é dada por 21 d(A, r).d(B, C). Temos que,
1 1
Area(ABC) = |(b1 c2 − b2 c1 ) − (a1 c2 − a2 c1 ) + (a1 b2 − a2 b1 )| = |∆|.
2 2
Q 1.8. Sejam r e s retas perpendiculares cujas equações são, respec-
tivamente, y = m1 x + a e y = m2 x + b. Mostre que m1 m2 = −1.

Solução: Reescrevendo a equação y = m1 x + a obtemos

m1 x − 1 · y = −a.

A última igualdade diz que (m1 , −1) é perpendicular à reta r. Então, o


vetor (1, m1 ) é paralelo a r. Analogamente, o vetor (m2 , −1) é perpen-
dicular à reta s. Portanto, os vetores (1, m1 ) e (m2 , −1) são paralelos.
Logo, existe λ ∈ R tal que (1, m1 ) = λ(m2 , −1). Segue que λm2 = 1 e
λ = −m1 . Donde, m1 m2 = −1.

Q 1.9. Mostre que as equações a1 x + b1 y = c1 e a2 x + b2 y = c2 re-


presentam duas retas paralelas se, somente se, existe λ ∈ R tal que
(a2 , b2 ) = λ(a1 , b1 ).

Solução: As equações a1 x + b1 y = c1 e a2 x + b2 y = c2 representam


duas retas paralelas se, somente se, seus vetores normais (a1 , b1 ) e
(a2 , b2 ) são paralelos, o que ocorre somente quando existe λ ∈ R tal
que (a2 , b2 ) = λ(a1 , b1 ).
38

1.10 Exercı́cios propostos

Q 1.10 (USJT-SP). Sabe-se que o ponto P = (4k − 1, 2k + 3) pertence


à bissetriz dos quadrantes ı́mpares. Determine o valor de k.

Q 1.11 (UECE-CE). Se (2, 5) é o ponto médio do segmento de ex-


tremos (5, y) e (x, 7), determine x + y.

Q 1.12 (PUC-SP). Um lado de um paralelogramo tem extremidades


nos pontos A = (−3, 5) e B = (1, 7). Sabendo que M = (1, 1) é o ponto
médio das diagonais, determine os outros vértices do paralelogramo.

Q 1.13 (ITA-SP). Os pontos A = (0, 0), B = (b, 2b) e C = (5b, 0), com
b > 0, são vértices de um retângulo. Determine o quarto vértice do
retângulo.

Q 1.14 (FUVEST-SP). Uma reta r determina, no primeiro quadrante do


plano cartesiano, um triângulo isósceles cujos vértices são a origem e
os pontos onde a reta intersecta os eixos. Se a área desse triângulo
é 18, determine a equação da reta r.

Q 1.15 (ESAM-RN). Determine a equação da reta que passa pelo


ponto (1, 0) e é perpendicular á reta de equação 2x − y = 7.

Q 1.16 (FUNESP-SP). Seja A a intersecção das retas r e s, de equa-


ções y = 2x e y = 4x − 2, respectivamente. Se B e C são as
intersecções das retas r e s com o eixo das abscissas. Determine
a área do triângulo ABC.

Q 1.17 (ITA). Qual o menor ângulo formado pelas retas de equações


3x − y = 10 e 2x + y = 6.

Q 1.18 (FURRN). Qual a soma dos valores de λ, tais que a reta de


π
equação 3x − λy = 10 forma um ângulo de 4
com a reta 2x + 5y = 17?

Q 1.19. Determine o ângulo formado pelas bissetrizes dos quadrantes


pares e ı́mpares.

Q 1.20. Qual a distância da origem à reta y = −x + 2 ?


39

Q 1.21 (UNIFOR-CE). As coordenadas de um ponto genérico de uma


2t−1
reta r são dados por x = 3
e y = t + 2, onde t é um parâmetro real.
Determine a equação da reta.

Q 1.22. Dados o ponto A = (1, 2) e a reta r de equação x − 2y = 3,


resolva os itens abaixo.

(a) Determine projr (A);

(b) d(A, r);

(c) O simétrico de A em relação a r é o ponto A′ tal que projr (A) é


ponto médio do segmento AA′ . Determine A′ .

Q 1.23 (URRN). Seja M o ponto de intersecção das retas de equações


x − y = 6 e 3x + y = 2. Obtenha a equação da reta paralela ao eixo
das abscissas passando por M.

Q 1.24. Determine a equação da reta que é paralela à reta de equação


8x − 2y = −1 e passa pelo ponto A = (3, −5).

Q 1.25. Prove que o segmento, cujas extremidades são os pontos


médios dos lados de um triângulo, é paralelo ao terceiro lado e igual
á metade deste.

Q 1.26. Demonstre que a medida da mediana relativa à hipotenusa é


igual à metade da medida da hipotenusa.

Q 1.27 (UECE-CE). Sejam A e B os pontos de intersecção dos gráficos


x2 +x
de f (x) = x2 − 3 e g(x) = 2
. Determine a medida do segmento AB.

Q 1.28 (FATEC). Três pontos A, B e C, pertencentes à reta r de equa-


ção y − 3x − 1 = 0, têm suas abscissas em progressão geométrica. O
ponto A é a intersecção de r com o eixo x, a ordenada de C é 8 e B
localiza-se entre A e C. Determine o ponto B.

Q 1.29 (CESGRANRIO). Obtenha a equação da reta que passa pelo


ponto (1, 1) e é perpendicular à reta de equação x + 3y = 5.
40

Q 1.30 (UFMG). Qual a relação existente entre m e n para que as


retas de equações 2x − my + 1 = 0 e nx + 3y + 5 = 0 sejam paralelas?

Q 1.31 (UFRS). Sabendo que as retas y = ax + 2 e y = (5 + 2b)x − 1


são paralelas e, ambas, perpendiculares à reta y = 2b x + 3, calcule o
valor a + b.

Q 1.32 (CESGRANRIO). Seja P o ponto de intersecção das retas r1 :


y = −3x + 3 e r2 : y = − x2 + 2 e, A e B, os pontos de intersecção
das retas r1 e r2 como o eixo x, respectivamente. Determine a área
do triângulo ABP .

Q 1.33 (UF-UBERLÂNDIA). Determine o valor de m para que a reta


de equação 2x + 3y + m = 0 forme com os eixos coordenados um
triângulo de área igual a 5.

Q 1.34. Prove que se os vértices de um triângulo são pontos cujas


coordenadas são números racionais, então a área deste triângulo é
um número racional.
41

1.11 Referências Bibliográficas

[1 ] BARSOTTI, L. Geometria analı́tica e vetores. 3a Ed. Local:


Artes Gráficas e Editora Unificado, 1984.

[2 ] BOYER, C. B. História da Matemática. 3a Ed. Local: Editora da


Universidade de São Paulo, 1974.

[3 ] CAMARGO, I. & BOULOS, P. Geometria Analı́tica - um trata-


mento vetorial. 3a Ed. Local: Editora Prentice Hall Brasil, 2005.

[4 ] CAROLI, A. J. de; CALLIOLI, C. A. & FEITOSA, M. O. Vetores,


Geometria Analı́tica: teoria e exercı́cios. 6a Ed. Local: Editora
Nobel, 1968.

[5 ] CAROLI, A.; CALLIOLI, C. & FEITOSA, M. Matrizes Vetores e


Geometria Analı́tica. 17a Ed. Local: Editora Nobel, 1984.

[6 ] IEZZI, G. Fundamentos de Matemática Elementar vol.7: geome-


tria analı́tica. Local: Atual Editora, 2001.

[7 ] LEHMANN, C. H. Geometria Analı́tica. 1a Ed. Local: México -


UTEHA, 1953.

[8 ] LIMA, E. L. Geometria Analı́tica e Álgebra Linear. Local: IMPA


- Coleção Matemática Universitária, 2001.

[9 ] MIDDLEMIS, R. R. Analytic Geometry. 2a Ed. Local: Editora


Mc Graw-Hill Book Company Inc, 1955.

[10 ] MURDOCH, D. C. Geometria Analı́tica: uma introdução ao cál-


culo vetorial e matrizes. 2aEd. Local: Editora Livros Técnicos e
Cientı́ficos, 1971.

[11 ] REIS, G. & SILVA, V. Geometria Analı́tica. 2a Ed. Local: Editora


LTC, 1996.

[12 ] SIMMONS, G. F. Cálculo com Geometria Analı́tica. 1a Ed. Lo-


cal: Editora Mc Graw-Hill, 1987.
42

[13 ] STEINBRUCH, A. & WINTERLE, P. Geometria Analı́tica. 2a Ed.


Local: Editora Makron, 1987.

[14 ] VENTURI, J. Cônicas e Quádricas. 5a Ed. Local: www.geometria


analitica.com.br, 2003.

Web-Bibliografia

[15 ]http://www.geometriaanalitica.com.br/index3.html

[16 ]http://www.paulomarques.com.br/arq6.htm

[17 ]http://pessoal.sercomtel.com.br/matematica/geometria/ganalitica
/ganalitica.htm

[18 ]http://www.ime.unicamp.br/ jardim/livro-GA.pdf

[19 ]http://www.geometriaanalitica.com.br

[20 ]www.mat.uc.pt/ picado/geomdif/


RESUMO UNIDADE 2

Nesta unidade, apresentamos a definição geométrica de cônicas


e obtemos equações analı́ticas que as representam. As equações
obtidas são equações do segundo grau a duas variáveis. Então surge
o questionamento: qual cônica uma equação (geral) do segundo grau
a duas variáveis representa? Nesta unidade, também apresentamos
ferramentas necessá- rias para respondermos a esta pergunta.

Finalizamos a unidade mostrando ao leitor situações cotidianas


onde as cônica - elipse, parábola e hipérbole - aparecem natural-
mente.
SUMÁRIO

(2.1) Introdução

(2.2) Circunferência

(2.3) Parábola

(2.4) Elipse

(2.5) Hipérbole

(2.6) Rotação de um conjunto

(2.7) Aplicação

(2.8) Nota histórica

(2.9) Exercı́cios resolvidos

(2.10) Exercı́cios propostos

(2.11) Referências Bibliográficas


2. Cônicas

2.1 Introdução

Figura 2.1: As cônicas - ver referência [14] pag. 119

Originalmente, as cônicas são as curvas de intersecção de certos


planos com a superfı́cie de um cone circular reto. Elas são: circun-
ferência, parábola, elipse e hipérbole. A circunferência é a intersecção
da superfı́cie do cone com um plano, que não passa pelo vértice e é

45
46

perpendicular a seu eixo. A parábola é a intersecção da superfı́cie do


cone com um plano paralelo a uma geratriz. A elipse é a intersecção
da superfı́cie do cone com um plano oblı́quo a seu eixo, não paralelo
a uma geratriz. A hipérbole é a intersecção da superfı́cie do cone com
um plano paralelo a seu eixo.

Neste capı́tulo, obteremos uma representação analı́tica para cada


cônica e apresentaremos um método para identificar que cônica uma
equação do segundo grau a duas variáveis representa.

2.2 Circunferência Alguns autores de-


nominam de cı́rculo,
A circunferência de centro P = (a, b) e raio r > 0 é o conjunto
de centro P e raio
formado pelos A = (x, y) tais que d(A, P ) = r.
r, o conjunto dos
pontos A tais que
................... P
d(A, P ) = r.
...........................

b r

Figura 2.2: Circunferência

Aplicando a fórmula da distância entre pontos temos que: um


ponto A = (x, y) pertence à circunferência de centro P = (a, b) e raio
r se, somente se,
(x − a)2 + (y − b)2 = r 2 .

Desenvolvendo a igualdade encontramos

x2 + y 2 − 2ax − 2by + (a2 + b2 − r 2 ) = 0.

Mais geralmente, mostra-se que o conjunto dos pontos que sa-


tisfazem a equação Ax2 + Bxy + Cy 2 + Dx + Ey + F = 0 é uma
circunferência se, somente se, A = C 6= 0, B = 0 e D 2 + E 2 > 4AF .
47

Provaremos que a condição apresentada é suficiente. De fato,


suponha que A = C 6= 0 (podemos considerar A = C > 0), B = 0
e D 2 + E 2 > 4AF . Então,

D E F
Ax2 + Cy 2 + Dx + Ey + F = 0 ⇒ x2 + y 2 + x + y + = 0.
A A A

Completando quadrados temos,


 D 2 D2  E 2 E2 F
x+ − 2
+ y+ − 2
+ = 0.
2A 4A 2A 4A A

Segue que,

 D 2  E 2  D 2 + E 2 − 4AF 2
x+ + y+ = .
2A 2A 2A

A igualdade encontrada representa a circunferência de centro e


raio dados, respectivamente, por
ANOTE: √
 D E D 2 + E 2 − 4AF
................................ P = − ,− ; r= .
2A 2A 2A
................................
Exemplo 2.2.1. A equação 2x2 +2y 2 −10x+6y−15 = 0 é a representa-
................................
ção analı́tica de uma circunferência. De fato, A = C = 2, B = 0 e
................................
√ p
................................ D 2 + E 2 − 4AF = (−10)2 + 62 − 4 · 2 · (−15)
√ √
................................ = 100 + 36 + 120 = 256 = 16.
................................  
5
Também segue de imediato que o centro da circunferência é 2
, − 23
e o raio é igual a 4.

2.3 Parábola

Sejam r uma reta e F = (m, n) 6∈ r um ponto. Chama-se parábola


de diretriz r e foco F o conjunto dos pontos X = (x, y) do plano que
satisfazem a equação

d(X, P ) = d(X, r),

isto é, o conjunto dos pontos do plano que são equidistantes do foco
e da diretriz.
48

foco

X | *F

|
diretriz − r

Figura 2.3: Parábola

Chamaremos de eixo da parábola a reta perpendicular à diretriz


passando no foco, e, de vértice a intersecção do eixo com a parábola.
Note que o vértice da parábola está no ponto médio do segmento de
reta que une o foco à intersecção do eixo com a diretriz.

eixo

* F

vértice

Figura 2.4: Elementos da Parábola

Vamos supor que a diretriz é paralela ao eixo x, ou seja, a reta


r tem equação y = −d. Aplicando a fórmula da distância entre dois
pontos e a fórmula da distância de um ponto a uma reta, concluı́mos
que: um ponto X = (x, y) pertence à parábola se, somente se,
p |y + d|
(x − m)2 + (y − n)2 = √ = |y + d|.
02 + 12
Como F = (m, n) 6∈ r temos que n 6= −d, isto é, n+d 6= 0. Podemos
supor sem perda de generalidade que n > −d. Neste caso, obteremos
uma parábola com a concavidade para cima, se supormos n < −d
obteremos uma parábola com a concavidade para baixo. Desenvol-
vendo a igualdade anterior obtemos,
1 2m (m2 + n2 − d2 )
y= x2 − x+ .
2(n + r) 2(n + r) 2(n + r)
Mais geralmente, parábolas ocorrem como gráficos de funções
quadráticas. Uma função quadrática de uma variável tem a forma
49

f (x) = ax2 + bx + c. O gráfico de f é o conjunto

G(f ) = {(x, y) ∈ R2 : y = ax2 + bx + c}.

Determinaremos, neste caso, o foco, o vértice e a diretriz da parábo-


la. Suponha que a equação y = ax2 + bx + c representa uma parábola
de foco F = (m, n) e diretriz y = −d. Segue que,
1 m m2 + n2 − d2
a= ; b=− ; c= .
2(n + d) n+d 2(n + d)
1
A primeira e a segunda igualdade implicam 2(n + d) = a
= − 2m
b
.
b
Logo, m = − 2a . Pela terceira igualdade temos,
m m (n + d)(n − d) bm n − d
c= · + =− + .
2 n+d 2(n + d) 2 2
1
Portanto, 2(n − d) = 2bm + 4c. Novamente utilizando que 2(n + d) = a
1 1+b2 −4ac
obtemos 4d = a
− 2bm − 4c. Donde, d = 4a
. Segue de imediato
4ac−b2 +1
que, n = 4a
. Isto diz que a diretriz da parábola tem equação
4ac−b2 −1 b 4ac−b +1 2
y= 4a
e o foco é F = (− 2a , 4a ).
Como o vértice está no ponto médio do segmento de reta que une
o foco à intersecção do eixo com a diretriz concluı́mos que o vértice é
b n−d
V = (− 2a , 2 ). Donde,
 b b2 − 4ac 
V = − ,− .
2a 4a
Exemplo 2.3.1. A parábola y = x2 − 8x + 15 tem foco no ponto (4, − 34 ),
a diretriz tem equação y = − 45 e o vértice é (4, 1). De fato, substi-
tuindo os valores a = 1, b = −8 e c = 15 nas fórmulas obtidas acima
Convidamos o
encontramos o resultado desejado.
leitor curioso a
determinar as Analogamente ao que vimos, se considerarmos a diretriz paralela
coordenadas do ao eixo y temos que a equação da parábola tem a forma geral
vértice da parábola
x = ay 2 + by + c.
de equação
x = ay 2 + by + c.
2.4 Elipse

Sejam F1 e F2 dois pontos distintos pertencentes a um plano e,


a > 0, um número real maior que a metade da distância entre F1 e F2 .
50

O conjunto dos pontos X tais que

d(X, F1 ) + d(X, F2 ) = 2a

chama-se elipse de focos F1 e F2 .

F1 F2

Figura 2.5: Elipse

A reta que passa nos focos chamaremos de eixo focal, e, os pon-


tos de intersecção do eixo focal com a elipse serão chamados de
vértices da elipse. Note que a distância entre os vértices é igual a
2a. Chamaremos a de semi-eixo focal da elipse.

Denominaremos por centro da elipse o ponto médio do segmento


de reta que une os focos; a mediatriz do segmento de reta que une
os focos será chamada de eixo normal da elipse. Seja b a metade da
distância entre os pontos de intersecção da elipse com o eixo normal.
Chamaremos b de semi-eixo normal. Se denotarmos por c a metade
da distância entre os focos, então, utilizando o Teorema de Pitágoras,
podemos concluir que a2 = b2 + c2 .

eixo normal
a
vértice b

F1 c F2

eixo focal

Figura 2.6: Elementos da elipse

Nesta secção, analisaremos o caso em que os focos pertencem


a uma reta paralela ao eixo x. Ou seja, vamos supor que os focos
são da forma F1 = (n1 , m) e F2 = (n2 , m), com n1 < n2 . Neste caso,
51

o centro da elipse é (x0 , y0) = ( n1 +n


2
2
, m) e n2 − n1 = 2c. Donde,
n2 −n1
x0 − n1 = 2
= c e n2 = 2x0 − n1 = x0 + c.
Pela definição temos que um ponto P = (x, y) pertence à elipse
se, somente se,
p p
(x − n1 )2 + (y − m)2 + (x − n2 )2 + (y − m)2 = 2a.

Portanto,
p p
(x − n1 )2 + (y − m)2 = 2a − (x − n2 )2 + (y − m)2 .

Elevando ao quadrado,
p
(x−n1 )2 +(y−m)2 = 4a2 −4a (x − n2 )2 + (y − m)2 +(x−n2 )2 +(y−m)2 .

Simplificando,
p
a [(x − x0 ) − c]2 + (y − y0 )2 = a2 − c(x − x0 ).

Novamente elevando ao quadrado e simplificando obtemos

(a2 − c2 )(x − x0 )2 + a2 (y − y0 )2 = a2 (a2 − c2 ).

A última igualdade implica


(x − x0 )2 (y − y0 )2
+ = 1.
a2 b2
A rigor, provamos acima apenas que as coordenadas (x, y) de um
ponto arbitrário da elipse satisfazem a equação encontrada. Mostra-
se, reciprocamente, que todo ponto cujas coordenadas satisfazem
esta equação pertence à elipse cujos focos são os pontos F1 = (n1 , y0)
Como estratégia
n1 +n2
e F2 = (n2 , y0 ), com x0 = 2
e n2 > n1 .
de aprendizagem,
2
x2
o leitor deve re- Exemplo 2.4.1. A equação 25
+ y16 = 1 representa a elipse centrada na
produzir as contas, origem, de semi-eixo focal 5, semi-eixo normal 4 e focos nos pontos
sem olhar as ante- (±3, 0).
riores, para o caso
Analogamente ao que vimos, se considerarmos os focos perten-
em que os focos
centes a uma reta paralela ao eixo y temos que a equação que repre-
pertencem a uma
senta a elipse é:
reta paralela ao (x − x0 )2 (y − y0 )2
+ = 1.
b2 a2
eixo y.
52

Chamaremos de excentricidade da elipse a razão entre c e a. Se


denotarmos por e a excentricidade, então e = ac . Note que a excen-
tricidade da elipse é um número situado entre zero e 1. No exemplo
anterior, a elipse tem excentricidade igual a 35 .

2.5 Hipérbole

Sejam F1 e F2 dois pontos do plano e a um número real positivo.


Chama-se de hipérbole de focos F1 e F2 ao conjunto dos pontos X
cuja diferença das distâncias aos pontos F1 e F2 é, em valor absoluto,
igual a 2a. Assim, o ponto X pertence a essa hipérbole se, somente
se,
|d(X, F1 ) − d(X, F2 )| = 2a.

O subconjunto da hipérbole formado pelos pontos X que satis-


fazem a igualdade d(X, F1 ) − d(X, F2 ) = 2a chamaremos de ramo da
hipérbole segundo F2 , e o subconjunto formado pelos pontos X tais
que d(X, F1 ) − d(X, F2 ) = −2a chamaremos de ramo da hipérbole se-
gundo F1 .

A reta que passa nos focos chamaremos de eixo focal e os pon-


tos de intersecção do eixo focal com a hipérbole serão chamados de
vértices da hipérbole. Note que a distância entre os vértices é igual a
2a. Chamaremos a de semi-eixo focal da hipérbole.
eixo normal

vértices

eixo focal b c
* * * *
F1 F2 * * a * *
F1 F2

focos

Figura 2.7: Elementos da hipérbole

Denominaremos por centro da hipérbole o ponto médio do seg-


mento de reta que une os focos; a mediatriz do segmento de reta que
53

une os focos será chamada de eixo normal da hipérbole. Se denotar-


mos por c a metade da distância entre os focos, temos c > a. Seja
b > 0 tal que c2 = a2 + b2 .

Vamos obter a equação reduzida da hipérbole quando os focos


são da forma F1 = (n1 , m) e F2 = (n2 , m), com n1 < n2 . Neste caso,
o centro da hipérbole é (x0 , y0) = ( n1 +n
2
2
, m) e n2 − n1 = 2c. Donde,
n2 −n1
x0 − n1 = 2
= c e n2 = 2x0 − n1 = x0 + c.

A fim de determinar a equação do ramo da hipérbole segundo F2 ,


escreveremos a equação d(X, F1 ) − d(X, F2 ) = 2a em termos de coor-
denadas, o que nos dá

p p
(x − n1 )2 + (y − m)2 − (x − n2 )2 + (y − m)2 = 2a.

Portanto,
p p
(x − n1 )2 + (y − m)2 = 2a + (x − n2 )2 + (y − m)2 .

Elevando ao quadrado,
p
(x−n1 )2 +(y−m)2 = 4a2 +4a (x − n2 )2 + (y − m)2 +(x−n2 )2 +(y−m)2 .

Simplificando,
p
−a [(x − x0 ) − c]2 + (y − y0 )2 = a2 − c(x − x0 ).

Novamente elevando ao quadrado e simplificando obtemos,

(a2 − c2 )(x − x0 )2 + a2 (y − y0 )2 = a2 (a2 − c2 ).

A última igualdade implica

(x − x0 )2 (y − y0 )2
− = 1.
a2 b2

Cálculos análogos aos apresentados acima nos conduzem à mes-


ma equação para o ramo da hipérbole segundo F1 .

A rigor, provamos acima apenas que as coordenadas (x, y) de


um ponto arbitrário da hipérbole satisfazem a equação encontrada.
54

Mostra-se, reciprocamente, que todo ponto cujas coordenadas satis-


fazem esta equação pertence à hipérbole cujos focos são F1 = (n1 , y0)
n1 +n2
e F2 = (n2 , y0 ), com x0 = 2
e n2 > n1 .

Exemplo 2.5.1. A equação 2x2 − 3y 2 = 5 equivale a

x2 y2
p − p = 1, ANOTE:
( 5/2)2 ( 3/2)2
................................
que representa a hipérbole centrada na origem, de semi-eixo focal
q q ................................
5 3
2
, semi-eixo normal 2
e de focos nos pontos (±2, 0).
................................
Analogamente ao que vimos, se considerarmos os focos perten- ................................
centes a uma reta paralela ao eixo y temos que a equação que repre- ................................
senta a hipérbole é: ................................
................................
(x − x0 )2 (y − y0 )2
− + = 1.
b2 a2

Chamaremos de excentricidade da hipérbole a razão entre c e a.


Se denotarmos por e a excentricidade, então e = ac . Note que a ex-
centricidade da hipérbole é um número maior 1. No exemplo anterior,
q q
a hipérbole tem excentricidade igual a 2 25 = 85 .

assíntotas

* * * *
F1 F2

centro

Figura 2.8: Assı́ntotas

As retas y = ± ab (x − x0 ) + y0 são chamadas de assı́ntotas da


hipérbole
(x − x0 )2 (y − y0 )2
− = 1.
a2 b2
Exemplo 2.5.2. As retas y = ± 34 x são as assı́ntotas da hipérbole
x2 y2
9
− 16
= 1.
55

2.6 Rotação de um conjunto

Na secção anterior obtivemos equações que representam as


cônicas, no caso em que os focos pertencem a uma reta paralela ao
eixo x ou ao eixo y. Mas isso nem sempre ocorre. Vejamos alguns
exemplos.

Exemplo 2.6.1. Seja P = (x, y) um ponto pertencente à parábola de


foco F = (1, 1) e diretriz r : x + y = 0. Então,
p |x + y|
d(P, F ) = d(P, r) ⇒ (x − 1)2 + (y − 1)2 = √ .
1+1

Desenvolvendo obtemos x2 − 2xy + y 2 − 4x − 4y + 4 = 0.

Exemplo 2.6.2. Considere a elipse de focos nos pontos (−3, 0) e


(0, 4), cujo semi-eixo focal mede 72 . Então, um ponto (x, y) pertence
à elipse se, somente se,
p p
(x + 3)2 + y 2 + x2 + (y − 4)2 = 7.

Efetuando cálculos, semelhantes aos apresentados para obtermos a


equação da elipse, encontramos que

40x2 − 24xy + 33y 2 + 168x − 168y − 200 = 0.


√ √
Exemplo 2.6.3. Considere a hipérbole de focos nos pontos (− 2, 2)
√ √ √
e ( 2, − 2), cujo semi-eixo focal mede 2. Então, um ponto (x, y)
pertence à hipérbole se, somente se,
q √ √ q √ √ √
(x + 2) + (y − 2) − (x − 2) + (y + 2)2 = 2 2.
2 2 2

Efetuando os cálculos obtemos a seguinte equação

xy = −1.

Nos três exemplos apresentados acima, as equações encontradas


são da forma

Ax2 + 2Bxy + Cy 2 + Dx + Ey + F = 0.
56

Portanto, é natural questionar que cônica uma equação geral da


forma Ax2 + 2Bxy + Cy 2 + Dx + Ey + F = 0 representa. Para res-
pondermos a esta pergunta necessitamos de alguns fatos, os quais
relembraremos a seguir.

O plano xy está em correspondência com o conjunto dos números


complexos, a saber: todo ponto (x, y) do plano cartesiano pode ser
visto como o número complexo x + iy, e vice-versa. Utilizaremos esta
correspondência para mostrarmos como rotacionar um conjunto em
relação à origem. O propósito de rotacionarmos um conjunto é trans-
formarmos a equação geral Ax2 + 2Bxy + Cy 2 + Dx + Ey + F = 0
numa equação do tipo Ar 2 + Ct2 + Dr + Et + F = 0, a qual possamos
comparar com as equações reduzidas, obtidas nas secções 2.2, 2.3 e
2.4, que representam as cônicas.

Necessitaremos de algumas propriedades dos números complexos.


Relembremos: dado um número complexo z = x + iy seu módulo é
p
definido como |z| = x2 + y 2. O argumento de z é o menor ângulo
x y
positivo θ tal que cos θ = √ e sin θ = √ . Usaremos a
x2 +y 2 x2 +y 2
seguinte notação arg(z) = θ.

Dados dois números complexos z1 = x1 + iy1 e z1 = x1 + iy1 as


seguintes propriedades são válidas:

1. |z1 · z2 | = |z1 | · |z2 |;

2. arg(z1 · z2 ) = arg(z1 ) + arg(z2 );

3. Observe também que | cos θ + i sin θ| = 1.

Das propriedades apresentadas acima concluı́mos que, para rotar-


cionarmos, de um ângulo θ, um conjunto sem alterarmos suas pro-
priedades basta multiplicarmos cada elemento deste conjunto pelo
complexo cos θ+i sin θ. Ou seja: se um ponto (x, y) = x+iy pertence a
um conjunto C, então o ponto (r, t) = r+it = (x+iy)(cos θ+i sin θ) per-
tence a um conjunto C ′ , com as mesmas propriedades de C, obtido
57

A grande maioria pela rotação de C, de um ângulo θ, no sentido anti-horário em torno


dos livros apresen- da origem.
tam uma abodagem
diferente da nossa.
Os autores, em (r,t)
(x,y)
geral, trabalham
0
com a mudança
dos eixos coorde- (0,0)

nados, ao invés
da rotação do con-
junto. Também
usaremos o método Figura 2.9: Rotação de um conjunto

da mudança dos
eixos coordena- Agora considere um ponto (x, y) que satisfaz a equação geral do
dos no capı́tulo segundo grau Ax2 + 2Bxy + Cy 2 + Dx + Ey + F = 0. Mostraremos
4, quando for- como obter o ângulo θ que transforma esta equação em uma do tipo
mos identificar as Ar 2 + Ct2 + Dr + Et + F = 0. Efetuando a multiplicação na igualdade
quádricas centrais. r + it = (x + iy)(cos θ + i sin θ) temos

 r = x cos θ − y sin θ
.
 t = x sin θ + y cos θ

Resolvendo o sistema encontrado obtemos as seguintes expressões


para x e y:

 x = r cos θ + t sin θ
.
 y = −r sin θ + t cos θ

Portanto,

φ(x, y) = Ax2 + 2Bxy + Cy 2 + Dx + Ey + F

= A(r cos θ + t sin θ)2 + C(−r sin θ + t cos θ)2

+ 2B(r cos θ + t sin θ)(−r sin θ + t cos θ)

+ D(r cos θ + t sin θ) + E(−r sin θ + t cos θ) + F.


58

Efetuando os cálculos,

φ(x, y) = Ax2 + 2Bxy + Cy 2 + Dx + Ey + F

= r 2 [A cos2 θ + C sin2 θ − B sin(2θ)]

+ t2 [A sin2 θ + C cos2 θ + B sin(2θ)]

+ rt[(A − C) sin(2θ) + 2B cos(2θ)]

+ r(D cos θ − E sin θ) + t(D sin θ + E cos θ) + F.

Dados A e C números reais, com A 6= C, existe um ângulo θ tal que


2B
tan(2θ) = − A−C . Donde, (A − C) sin(2θ) + 2B cos(2θ) = 0. Se A = C,
π
escolhendo θ = 4
também obtemos (A − C) sin(2θ) + 2B cos(2θ) = 0.
Assim, sempre existe θ ∈ R tal que (A − C) sin(2θ) + 2B cos(2θ) = 0.
Resulta que,

φ(x, y) = Ax2 + 2Bxy + Cy 2 + Dx + Ey + F

= r 2 [A cos2 θ + C sin2 θ − B sin(2θ)]

+ t2 [A sin2 θ + C cos2 θ + B sin(2θ)]

+ r(D cos θ − E sin θ) + t(D sin θ + E cos θ) + F.

Consequentemente, a equação Ax2 +2Bxy+Cy 2 +Dx+Ey+F = 0


é equivalente a

ϕ(r, t) = r 2 [A cos2 θ + C sin2 θ − B sin(2θ)]

+ t2 [A sin2 θ + C cos2 θ + B sin(2θ)]

+ r(D cos θ − E sin θ) + t(D sin θ + E cos θ)

+ F = 0. ANOTE:
................................
Perceba que a nova equação obtida não envolve o termo rt. Com
................................
isto, podemos completar quadrados e compararmos ϕ(r, t) = 0 com
................................
as equações reduzidas que representam as cônicas.
................................
Exemplo 2.6.4 (Parábola). Na equação 2x2 +12xy +18y 2 +x+y +1 = 0 ................................
temos A = 2, B = 6, C = 18 e D = E = F = 1. Seja θ tal que ................................
................................
(A − C) sin(2θ) + 2B cos(2θ) = −16 sin(2θ) + 12 cos(2θ) = 0.
59

Portanto, 4 sin(2θ) = 3 cos(2θ). Pela relação fundamental da trigonome-


tria, sin2 (2θ) + cos2 (2θ) = 1, obtemos sin(2θ) = 3
5
e cos(2θ) = 45 . Agora
1−cos(2θ) 1+cos(2θ)
usando as identidades sin2 θ = 2
e sin2 θ = 2
, encon-
tramos sin2 θ = 1
10
e cos2 θ = 9
10
. Donde,

0 = 2x2 + 12xy + 18y 2 + x + y + 1


9 1 3
= r 2 (2. + 18. − 6. )
10 10 5
2 1 9 3
+ t (2. + 18. + 6. )
10 10 5
3 1 3 1
+ r(1. √ − 1. √ ) + t(1. √ + 1. √ ) + 1.
10 10 10 10
√ 2 √
Segue que, r = −10 10t − 2t − 210 . A última equação obtida repre-
senta uma parábola.

Exemplo 2.6.5 (Elipse). Dado F ∈ R, identificaremos que cônica a


equação 5x2 + 6xy + y 2 = F representa. Note que A = C = 5 e B = 3.

2
Neste caso, θ = π4 . Então, sin θ = cos θ = 2
e sin(2θ) = 1. Segue que,

F = 5x2 + 6xy + y 2
1 1 1 1
= r 2 (5. + 5. − 3.1) + t2 (5. + 5. + 3.1)
2 2 2 2
2 2
= 2r + 8t .

Temos três possibilidades a considerar. F < 0 : Nesta caso, a equação


2r 2 +8t2 = F representa um conjunto vazio, pois não existem números
maiores ou iguais a zero cuja soma seja um número negativo. F = 0 :
Nesta possibilidade, a equação 2r 2 +8t2 = F representa o ponto (0, 0).
F > 0 : Observe que a equação 2r 2 + 8t2 = F é equivalente a
r2 t2
q 2 q 2 = 1,
+
F F
2 8
q
F
que representa uma elipse centrada na origem e semi-eixo focal 2
.

No exemplo anterior foi observado que nem sempre uma equação


geral Ax2 + 2Bxy + Cy 2 + Dx + Ey + F = 0 representa um cônica. A
mesma pode representar um conjunto vazio, um ponto, uma reta ou
um par de retas. Como saber? Fazendo a rotação, de um ângulo θ,
de modo a eliminar o termo 2Bxy e completar quadrados.
60

Exemplo 2.6.6 (Hipérbole). Na equação xy = 1 temos A = C = 0,


π
2B = 1, D = E = 0 e F = −1. Sendo A = C temos θ = 4
. Então,

2
sin θ = cos θ = 2
e sin(2θ) = 1. Segue que,

1 1
1 = xy = − r 2 + t2
2 2
r2 t2
= − √ + √ .
( 2)2 ( 2)2
2 2
A equação − (√r2)2 + √t
( 2)2
= 1 representa uma hipérbole de focos nos
pontos (0, ±2).

2.7 Aplicação

Parábola

Por que antenas que captam mais sinais do espaço são parabólicas?
Por que os espelhos dos telescópios astronômicos são parabólicos?

Figura 2.10: Antena e Espelho parabólicos

Nos dois exemplos acima, os sinais que recebemos (ondas de


rádio ou luz) são muito fracos. Por isso é necessário captá-los em
um único ponto para que sejam naturalmente amplificados. Portanto,
a superfı́cie da antena ( ou do espelho) deve ser tal que todos os sinais
61

recebidos de uma mesma direção sejam direcionados para um único


ponto após a reflexão.

A parábola possui exatamente esta propriedade e, por isso, as an-


tenas e os espelhos precisam ser parabólicos.

Elipse

A trajetória dos planetas ao redor do Sol não é circular e sim


elı́ptica - o Sol fica sobre um dos focos da elipse. Foi Kepler (1571-
1630) quem desenvolveu esta teoria. No caso da Terra os semi-eixos
são a = 153.493.000km e b = 153.454.000km. Donde podemos obter a
excentricidade da órbita da Terra:
c
e= = 0, 0167.
a
O eixo maior apresenta dois pontos: o periélio (janeiro) e o afélio
(julho), que correspondem às distâncias mı́nimas e máximas da Terra
ao Sol, respectivamente.

Hipérbole

O sistema LORAN (longe range navegation) de navegação aérea


usa a hipérbole. Da Terra, concomitantemente são transmitidos sinais
de rádio de dois pontos fixos F1 e F2 que são captados pelo aeroplano
em P , ao longo de t1 e t2 segundos, respectivamente. A diferença en-
tre t1 e t2 determina 2a e assim obtém-se a caracterı́stica da hipérbole
na qual está P .

2.8 Nota histórica

Menaecmus, Apolônio a as secções cônicas

No século IV a.C., quando violenta peste assolava Atenas, os


morado- res da cidade resolveram aconselhar-se com o oráculo de
62

Delos. Este lhes sugeriu que o altar a Apolo na ilha, que era cúbico,
deveria ser dobrado. Os atenienses se apressaram em construir um
outro, com o dobro das dimensões do anterior. Ora, se as dimensões
do altar mediam x e passaram a medir 2x, o volume do altar passou
de x3 para 8x3 - ou seja, octoplicou. Consta que a peste se inten-
sificou e os atenienses decidiram, então, consultar os matemáticos
da academia de Platão sobre como determinar as dimensões que
atendessem à recomendação do oráculo. Estava surgindo assim o
problema da duplição do cubo que, no fundo, consiste em construir

um segmento de reta cuja medida seja 3 2.

Menaecmus, um discı́pulo de Eudóxio e membro da academia de


Platão, ocupa um lugar especial entre os matemáticos que se pro-
puseram a resolver esse problema. É que, além de lograr êxito em
sua empreitada, o caminho que tomou propiciou-lhe a descoberta das
secções cônicas: elipse, parábola e hipérbole. Em notação moderna
é fácil concluir que a intersecção da parábola y = x2 com a hipérbole

xy = 2 é o ponto de abscissa 3 2.

Mas Menaecmus não dispunha do recurso de uma notação algébri-


ca, posto que a matemática de sua época era essencialmente geomé-
trica. E introduziu essas curvas usando três tipos de superfı́cies côni-
cas ilimitadas de uma folha: com secção meridiana aguda, reta ou
obtusa. Interceptando cada superfı́cie dessas com um plano perpen-
dicular a uma de suas secções meridianas, obtinha respectivamente
uma elipse, uma parábola e uma hipérbole. Fica evidente a partir
desta definição o porquê da designação secções cônicas para essas
curvas. Mas os nomes elipse, parábola e hipérbole seriam introduzi-
dos por Apolônio de Perga (c. 262-190 a.C.).

Natural de Perga, colônia grega ao sul da Ásia Menor, Apolônio es-


tudou matemática em Alexandria, onde passou também algum tempo
ensinando. Ensinou em Pérgamo, cuja biblioteca, na época, somente
era excedida pela de Alexandria. Conhecido como ”o grande geômetra”,
63

sua obra maior é Secções cônicas, em oito livros, dos quais restaram
apenas os sete primeiros.

Ao contrário de Menaecmus, Apolônio obtinha todas as secções


cônicas numa única superfı́cie cônica circular genérica de duas folhas,
mediante inclinações convenientes dos planos de secção.

Assim, através de um estudo integrado dessas curvas, conseguiu


em sua obra, resultados de grande alcance e originalidade. De fato,
Secções cônicas, em suas 487 proposições, praticamente esgota o
assunto sob o ponto de vista teórico e, com justa razão, é considerado
o ponto alto da geometria grega.

Dente os livros, o mais notável talvez seja o quinto, cujo objetivo


é a determinaç ão de distâncias máximas ou mı́nimas de um ponto
particular a uma cônica.

Apolônio buscou os nomes elipse, parábola e hipérbole na matemá-


tica da escola pitagórica, no problema da aplicação das áreas em
suas três formas: por falta (elipse), simples (parábola) e por excesso
(hipérbole). De fato, em coordenadas as três curvas admitem uma
equação geral da forma y 2 = 2px + qx2 (p > 0). O caso q < 0 fornece
elipses; o caso q = 0 fornece parábolas; e q > 0, hipérboles. Embora
sem usá-las, Apolônio tinha o sentido das coordenadas.

Hygino H. Domingues

2.9 Exercı́cios resolvidos

Q 2.1. Determine p de modo que o ponto P = (7, 9) seja exterior à


circunferência de equação x2 + y 2 − 2x − 2y − p = 0.

Solução: Para que a equação represente uma circunferência deve-


mos ter (−2)2 + (−2)2 − 4(−p) > 0. Donde, p > −2. O centro da
64

circunferência é C = (− (−2)
2
, − (−2)
2
) = (1, 1), e, o raio é

8 + 4p p
r= = 2 + p.
2
O ponto P é exterior à circunferência se, somente se, d(P, C) > r.
p √
Assim, (7 − 1)2 + (9 − 1)2 > 2 + p ⇒ 100 > 2 + p ⇒ 98 > p.
Portanto, −2 < p < 98.

2
Q 2.2. Qual o comprimento da corda que a reta R : x = 2
r determina
na circunferência x2 + y 2 = r 2 .

...............

L=2.l
r
...............

Figura 2.11: Corda determinada por R

Solução: É fácil ver que o centro da circunferência é O = (0, 0). Cal-



2
culemos a distância do centro da circunferência à reta x = 2
r:
√ √
| − 22 r| 2
d(O, R) = √ = r.
0+1 2
Seja L = 2l o comprimento da corda determinada por R, então
 √ 2 √ √
r 2 = 22 r + l2 ⇒ l = 22 r ⇒ L = 2r.

Q 2.3. Determine c de modo que a reta R : 4x−3y + c = 0 seja exterior


à circunferência de equação x2 + y 2 − 2x − 2y + 1 = 0.

Solução: Completanto quadrados obtemos que a equação da circun-


ferência é equivalente a (x − 1)2 + (y − 1)2 = 1. Então, o centro é
C = (1, 1) e o raio é r = 1.
A reta R é exterior à circunferência se, somente se, d(C, R) > r.
Donde,
|4 − 3 + c| |1 + c|
1< p = ⇒ c < −6; c > 4.
42 + (−3)2 5
65

Q 2.4. Determine a equação da circunferência que passa pelos pontos


A = (4, 2), B = (−1, 1) e D = (1, −1).

Solução: O centro da circunferência C = (x, y), que pertence à me-


diatriz de AB, é equidistante de A = (4, 2) e B = (−1, 1), ou seja,
d(C, A) = d(C, B). Donde,

(x − 4)2 + (y − 2)2 = (x + 1)2 + (y − 1)2 ⇒ 5x + y = 9.

O ponto C = (x, y) também pertence à mediatriz de BD, logo


d(C, D) = d(C, B). Assim,

(x − 1)2 + (y + 1)2 = (x + 1)2 + (y − 1)2 ⇒ x − y = 0.

Resolvendo o sistema formado pelas duas equações encontradas


obtemos x = y = 23 . Então, C = ( 32 , 23 ). O raio é dado por:

r r r
3 2  3 2 25 1 13
r = d(A, C) = 4− + 2− = + = .
2 2 4 4 2
 2  2
Portanto, a equação da circunferência é x − 23 + y − 32 = 13 2
.

Q 2.5. Encontre os focos, o centro, a excentricidade e o semi-eixo


focal da hipérbole xy = 1.

2
Solução: Como A = C = 0, então θ = π4 . Donde, sin θ = cos θ = 2
,
sin(2θ) = 1 e

 √ √
 x = 2 2
2
r + 2
t
√ √ .
 y = − 2r + 2t
2 2

2 2
Portanto, a equação xy = 1 é equivalente a − (√r2)2 + (√t2)2 = 1, que
representa uma hipérbole. Para a nova hipérbole temos a seguinte

relação a = b = 2 ⇒ c = 2. Como a primeira coordenada do par
(r, t) aparece com o sinal de menos, temos que os focos da hipérbole
pertencem ao eixo y. Daı́, os focos são F1 = (0, −2) e F1 = (0, 2), e, o
centro é o ponto C = (0, 0).
66

Para encontrarmos os elementos da hipérbole de coordenadas x e


y basta utilizarmos o sistema que relaciona as coordenadas das duas
hipérboles. Donde, C = (0, 0) é o centro da hipérbole xy = 1, e seus
√ √ √ √
focos são F1 = (− 2, − 2) e F2 = ( 2, 2). A excentricidade é

e = ac = 2.

2.10 Exercı́cios propostos

Q 2.6. Para quais valores de m, n e p a circunferência, cuja representa-


ção analı́tica é x2 + y 2 − mx − ny + p = 0, tem centro na origem?

Q 2.7. Qual a posição relativa (interior, pertencente ou exterior) do



ponto (1, 2) em relação à circunferência x2 + y 2 − 4x − 4y + 4 = 0?

Q 2.8. Quais são as equações das retas paralelas ao eixo x e tan-


gentes à circunferência (x + 2)2 + (y + 1)2 = 16?

Q 2.9. Determine a equação da reta que passa pelo centro da circun-


ferência de equação x2 + y 2 − 4x + 2y + 1 = 0 e é perpendicular à reta
de equação x + 2y = 14.

Q 2.10. Qual é o comprimento da corda que a reta 7x − 24y = 4


determina na circunferência x2 + y 2 − 2x + 6y = 15?

Q 2.11. Encontre a equação da circunferência que passa pelos pontos


(0, 1) e (1, 4) e tem centro sobre a reta x = 2.

Q 2.12. Determine a equação da circunferência que tem centro na


reta x − y = 4 e passa pelos pontos (0, −2) e (2, 0).

Q 2.13. Mostre que a reta y = ax + b é tangente á circunferência


x2 + y 2 = r 2 se, somente se, b2 = (1 + a2 )r 2 .

Q 2.14. Ache as equações das retas que passam pelo ponto (2, 5) e
são tangentes à circunferência de centro (0, 0) e raio 3.

Q 2.15. Completando quadrados, decida se cada uma das equações


abaixo define uma circunferência, um ponto ou o conjunto vazio.
67

• 2x2 + 2y 2 − 3x + y − 1 = 0;

• −x2 − y 2 + 6x − 4y + 3 = 0;

• x2 + y 2 − 10x + 2y + 26 = 0;

• 4x2 + 4y 2 − 4x − 8y + 21 = 0.

Q 2.16. Determine as coordenadas do vértice da parábola cuja equa-


ção é 2x2 + 4x + 3y − 4 = 0.

Q 2.17. Determine a equação da parábola cuja diretriz é x = 0 e o


foco é F = (4, 1).

Q 2.18. Qual a equação da mediatriz do segmento cujas extremidades


são os vértices das parábolas y = x2 + 6x + 4 e y = x2 − 6x + 2.

Q 2.19. Diz-se que uma reta é tangente a uma parábola quando tem
um único ponto em comum com ela. Determine a e b de modo que a
reta y = ax + b seja tangente à parábola y = x2 − 2x + 5.

Q 2.20. O ponto C = (4, 3) é o centro de uma elipse tangente aos


eixos cartesianos. Se o eixo focal é paralelo a um dos eixos coorde-
nados, escreva a equação da elipse.

Q 2.21. Determine o centro, os focos, o semi-eixo focal, o semi-eixo


normal e a excentriciade da elipse

(x − 2)2 (y − 3)2
+ = 1.
4 16

Q 2.22. Obtenha a equação reduzida da elipse cujo eixo normal tem


por extremos os focos da hipérbole 9x2 − 16y 2 = −144 e cuja excentri-
cidade é o inverso da excentricidade da hipérbole dada.

Q 2.23. Em cada um dos itens abaixo verifique qual cônica a equação


representa e obtenha seus elementos: foco(s), semi-eixo focal, centro,
excentricidade e diretriz.

• 4x2 + 4xy + y 2 + x − 2y = 0;
68

• 60x2 − 16xy + 48y 2 − 132x − 264y + 319 = 0;


√ √
• 3x2 − 10xy + 3y 2 + 12 2x − 4 2y + 32 = 0.

Q 2.24 (UECE-CE). Sejam M = (7, −2) e N = (5, 4). Se C é uma


circunferência que tem o segmento MN como diâmetro, determine a
equação de C.

Q 2.25 (MACK-SP). Determine o maior inteiro k para que a equação


x2 + y 2 + 4x − 6y + k = 0 represente uma circunferência.

3
Q 2.26 (FUVEST-SP). A reta de equação y = 3
x é tangente a uma
circunferência de centro (2, 0). Qual a equação dessa circunferência?

Q 2.27 (UFSM-RS). Obtenha a equação da circunferência que passa


pelos pontos (0, 0), (1, 0) e (0, 1).

Q 2.28 (UFPE-PE). Qual a equação da circunferência de centro (4, 4)


e que é tangente à reta x − y + 4 = 0?

Q 2.29 (FUVEST-SP). Uma reta passa pelo ponto (3, 1) e é tangente


à circunferência, de centro (1, 1) e raio 1, num ponto T . Determine a
medida do segmento P T .

Q 2.30 (ITA-SP). Sabendo que o ponto (2, 1) é ponto médio de uma


corda AB da circunferência (x − 1)2 + y 2 = 4, determine a equação da
reta que contém A e B.

Q 2.31 (UFMA-MA). Qual a equação da circunferência concêntrica


com a circunferência x2 + y 2 − 2x − 8y + 1 = 0 e tangente à reta
2x − y = 3?

Q 2.32 (FGV-SP). Uma parábola de equação y = ax2 + bx + c passa


pela origem do sistema de coordenadas e é tangente à reta y = 4, no
ponto (2, 4). Determine o valor de a + b + c.

Q 2.33 (PUC-SP). Determine as coordenadas do vértice da parábola


2x2 + 4x + 3y − 4 = 0.
69

Q 2.34 (PUC-SP). A equação 9x2 + 4y 2 −18x−16y −11 = 0 representa


uma elipse. Qual a medida do semi-eixo focal e do semi-eixo normal?
2
x2
Q 2.35 (PUC-SP). Um ponto P de elipse 9
+ y4 = 1 dista 2 de um dos
focos. Qual a distância de P ao outro foco?
70

2.11 Referências Bibliográficas

[1 ] BARSOTTI, L. Geometria analı́tica e vetores. 3a Ed. Local:


Artes Gráficas e Editora Unificado, 1984.

[2 ] BOYER, C. B. História da Matemática. 3a Ed. Local: Editora da


Universidade de São Paulo, 1974.

[3 ] CAMARGO, I. & BOULOS, P. Geometria Analı́tica - um trata-


mento vetorial. 3a Ed. Local: Editora Prentice Hall Brasil, 2005.

[4 ] CAROLI, A. J. de; CALLIOLI, C. A. & FEITOSA, M. O. Vetores,


Geometria Analı́tica: teoria e exercı́cios. 6a Ed. Local: Editora
Nobel, 1968.

[5 ] CAROLI, A.; CALLIOLI, C. & FEITOSA, M. Matrizes Vetores e


Geometria Analı́tica. 17a Ed. Local: Editora Nobel, 1984.

[6 ] IEZZI, G. Fundamentos de Matemática Elementar vol.7: geome-


tria analı́tica. Local: Atual Editora, 2001.

[7 ] LEHMANN, C. H. Geometria Analı́tica. 1a Ed. Local: México -


UTEHA, 1953.

[8 ] LIMA, E. L. Geometria Analı́tica e Álgebra Linear. Local: IMPA


- Coleção Matemática Universitária, 2001.

[9 ] MIDDLEMIS, R. R. Analytic Geometry. 2a Ed. Local: Editora


Mc Graw-Hill Book Company Inc, 1955.

[10 ] MURDOCH, D. C. Geometria Analı́tica: uma introdução ao cál-


culo vetorial e matrizes. 2aEd. Local: Editora Livros Técnicos e
Cientı́ficos, 1971.

[11 ] REIS, G. & SILVA, V. Geometria Analı́tica. 2a Ed. Local: Editora


LTC, 1996.

[12 ] SIMMONS, G. F. Cálculo com Geometria Analı́tica. 1a Ed. Lo-


cal: Editora Mc Graw-Hill, 1987.
71

[13 ] STEINBRUCH, A. & WINTERLE, P. Geometria Analı́tica. 2a Ed.


Local: Editora Makron, 1987.

[14 ] VENTURI, J. Cônicas e Quádricas. 5a Ed. Local: www.geometria


analitica.com.br, 2003.

Web-Bibliografia

[15 ]http://www.geometriaanalitica.com.br/index3.html

[16 ]http://www.paulomarques.com.br/arq6.htm

[17 ]http://pessoal.sercomtel.com.br/matematica/geometria/ganalitica
/ganalitica.htm

[18 ]http://www.ime.unicamp.br/ jardim/livro-GA.pdf

[19 ]http://www.geometriaanalitica.com.br

[20 ]www.mat.uc.pt/ picado/geomdif/


72
RESUMO UNIDADE 3

Por sua importância, utilidade e simplicidade conceitual, os sis-


temas lineares figuram nos currı́culos das escolas, a partir do segundo
grau. Entretanto, as exposições elementares do assunto costumam
deixar de lado seu significado geométrico. As incógnitas x, y, z, de
um sistema 3 × 3, são as coordenadas de um ponto no espaço de
três dimensões e cada uma das equações representa um plano nesse
espaço.

Para possibilitar essa abordagem geométrica, faremos uma apre-


sentação da Geometria Analı́tica no espaço, sempre fazendo uso da
intuição geométrica que temos sobre o espaço que nos rodeia. Con-
cluı́mos a unidade mostrando, por meio de um exemplo, onde apli-
camos a teoria estudada no nosso dia a dia.
SUMÁRIO

(3.1) Introdução

(3.2) Sistema de coordenadas no espaço

(3.3) Distância entre pontos no espaço

(3.4) Vetores no espaço

(3.5) Produto vetorial

(3.6) Equação do plano

(3.7) Distância de um ponto a um plano

(3.8) Retas reversas

(3.9) Aplicação

(3.10) Exercı́cios resolvidos

(3.11) Exercı́cios propostos

(3.12) Referências Bibliográficas


74
3. O espaço

3.1 Introdução

Neste capı́tulo, introduziremos um sistema de coordenadas carte-


sianas no espaço euclidiano tridimensional.

A introdução de coordenadas no espaço oferece não apenas um


método para resolver problemas geométricos com os recursos da Álge-
bra como, reciprocamente, fornece uma interpretação geométrica valio-
sa para questões de natureza algébrica, como o estudo dos sistema
lineares.

3.2 Sistema de coordenadas no espaço

y
O

Figura 3.1: Sistema de coordenadas no espaço

Considere um plano qualquer no espaço e, neste plano, um sis-


tema de coordenadas xy. Seja r a reta perpendicular a esse plano

74
75

passando na origem O do sistema xy. Em r, considere um sistema de


coordenadas tal que O também seja a origem.

A partir do sistema xy, no plano, e do sistema de coordenadas em


r, estabeleceremos um sistema de coordenadas para o espaço. Colo-
caremos o espaço em correspondência biunı́voca com o conjunto R3 ,
definido como R3 = {(x, y, z) : x, y, z ∈ R}. Dado um ponto P no
espaço, consideremos a reta passando por P que é perpendicular ao
plano xy, seja P ′ o ponto de intersecção. Sejam x e y, respectiva-
mente, a abscissa e a ordenada do ponto P ′ no plano xy, e z a coor-
denada, em r, da intersecção de r com sua perpendicular que passa
por P . Associamos o ponto P ao trio ordenado (x, y, z). As compo-
nentes x, y e z do trio (x, y, z) serão chamadas de coordenadas de P .
A correspondência assim definida é biunı́voca.

z
Durante todo o livro, P=(x,y,z)
*
vamos considerar y
O
apenas eixos or-
x
togonais (sistema
de coordenandas
cartesianas), mas é Figura 3.2: Coordenadas de um ponto
possı́vel adotar no
espaço Euclidiano
sistemas de eixos
oblı́quos. Basta A correspondência que acabamos que descrever chama-se um

tomar eixos x, y e sistema de coordenandas cartesianas para o espaço. Usaremos a


z com a mesma notação P = (x, y, z) para simbolizar que o ponto P tem coordenadas
origem e que não (x, y, z). Comumente, a reta r é chamada de eixo vertical, ou eixo dos
estão situados os z ou eixo z. Também é comum chamar o sistema de coordenadas
três no mesmo acima descrito de sistema de coordenadas xyz.
plano.
76

3.3 Distância entre pontos no espaço

Sejam A = (a1 , a2 , a3 ) e B = (b1 , b2 , b3 ) pontos do espaço, considere


os pontos auxiliares Q = (a1 , b2 , b3 ) e P = (a1 , a2 , b3 ).

P Q y

Figura 3.3: Distância entre dois pontos

Aplicando o Teorema de Pitágoras, duas vezes, obtemos que a


distância ente A e B á dada por

d2 (A, B) = d2 (P, A) + d2 (P, B) = d2 (P, A) + d2 (Q, P ) + d2 (Q, B).

Substituindo as coordenadas de cada ponto podemos concluir que:


p
d(A, B) = (a1 − b1 )2 + (a2 − b2 )2 + (a3 − b3 )2 .

Exemplo 3.3.1. A distância entre os pontos A = (1, 2, 3) e B = (2, 3, 4)


é igual a
p
d(A, B) = (1 − 2)2 + (2 − 3)2 + +(3 − 4)2
p √
= (−1)2 + (−1)2 + (−1)2 = 3.

Uma esfera de centro C = (x0 , y0, z0 ) e raio r > 0 é o conjunto de


pontos P = (x, y, z) do espaço tais que

d(P, C) = r.
p
Como d(P, C) = (x − x0 )2 + (y − y0 )2 + (z − z0 )2 , temos que um
ponto P = (x, y, z) pertence à esfera de centro C = (x0 , y0 , z0 ) e raio
r > 0 se, somentes se,

(x − x0 )2 + (y − y0 )2 + (z − z0 )2 = r 2 .
77

Exemplo 3.3.2. A equação x2 + y 2 + z 2 + 2x − y = 1 representa uma

ANOTE: esfera de centro no ponto C = (−1, 21 , 0) e raio r = 23 . De fato, comple-

................................ tando quadrados temos que:


................................ 1 1 9  3 2
(x + 1)2 + (y − )2 + z 2 = 1 + 1 + = = .
................................ 2 4 4 2

................................
Definição 3.3.1. O comprimento do segmento de extremidades A e B
................................
é definido como a distância de A a B.
................................
................................ Definição 3.3.2. Chamamos de ponto médio do segmento AB o ponto
M deste segmento tal que d(A, M) = d(M, B).

Proposição 3.3.3. O ponto médio do segmento de reta cujas extremi-


dades são os pontos A = (a1 , a2 , a3 ) e B = (b1 , b2 , b3 ) é o ponto
a + b a + b a + b 
1 1 2 2 3 3
M= , , .
2 2 2

Demonstração. Um cálculo simples mostra que d(A, M) = d(M, B), o


que significa que M é o ponto médio de AB.

3.4 Vetores no espaço

No cap.1, definimos um vetor como sendo um par ordenado de


núme- ros reais. Esta definição foi motivada pelo fato de que a cada
par (x, y) podemos fazer corresponder um vetor. Fato semelhante
verifica-se no espaço.

Figura 3.4: Vetor

Dados A, B ∈ R3 , chama-se vetor com ponto inicial em A e ponto


−→
final em B, denotado por AB, o segmento orientado no sentido de A
para B. Veja sua representação geométrica na figura 3.4.
78

Dada uma terna (x, y, z). Além de ser interpretado como as co-
ordenadas de um ponto do espaço, ela também pode ser vista como
um vetor cujo ponto inicial é a origem do sistema de coordenadas e o
ponto final é o ponto P = (x, y, z) correspondente a tais coordenadas.

z
P=(x,y,z)
*
y
O

Figura 3.5: Coordenadas de um vetor

Sejam u = (x1 , x2 , x3 ) e v = (y1 , y2 , y3) vetores e λ ∈ R. Então a


soma de vetores, o produto de um número por um vetor, o produto
interno de dois vetores e a norma de um vetor são definidos, respec-
tivamente, por:

• u + v = (x1 , x2 , x3 ) + (y1 , y2 , y3) = (x1 + y1 , x2 + y2 , x3 + y3 )

• λ · u = λ(x1 , x2 , x3 ) = (λx1 , λx2 , λx3 )

• hu, vi = h(x1 , x2 , x3 ), (y1, y2 , y3 )i = x1 y1 + x2 y2 + x3 y3


p
• |u| = |(x1 , x2 , x3 )| = x21 + x22 + x23

As definições apresentadas são análogas às suas correspondentes


para vetores no plano. Todas as propriedades enunciadas, no cap.1,
para estas operações, continuam válidas aqui. Ressaltamos a vali-
dade de uma delas, a desigualdade de Cauchy-Schwarz: Dados u, v ∈
R3 , então
|hu, vi| ≤ |u| · |v|.
É importante que o
Na desigualdade de Cauchy-Schwarz ocorre a igualdade somente leitor esteja familia-
quando um dos vetores é múltiplo do outro. rizado com os resul-
tados e notações do
Além das definições listadas acima, todas as propriedades rela-
capı́tulo 1.
cionadas e quaisquer outras definições recorrentes também são váli-
79

das. Como por exemplo, se θ é o ângulo formado pelos vetores u e v,


então
hu, vi
cos θ = .
|u| · |v|

v
0

Figura 3.6: Ângulo entre vetores

Deduzimos que dois vetores u e v são perpendiculares se, so-


mente se,
hu, vi = 0.

3.5 Produto vetorial

Dados dois vetores em R3 , digamos u = (a1 , a2 , a3 ) e v = (b1 , b2 , b3 ),


queremos encontrar um vetor não nulo w, em R3 , tal que w seja per-
pendicular a u e v. Para isto devemos ter, hu, wi = 0 e hv, wi = 0.
Se w = (x, y, z), então:

 a x+a y+a z = 0
1 2 3
.
 b x+b y+b z = 0
1 2 3

Tentaremos resolver o sistema encontrado. Para isto, multiplicare-


mos a primeira equação por b2 , a segunda por −a2 e, em seguida,
somaremos as duas equações. Obteremos a seguinte equação:

(a1 b2 − a2 b1 )x = (a2 b3 − a3 b2 )z.

Agora, multiplicando a primeira equação do sistema por −b1 , a se-


gunda por a1 e, em seguida, somando as equações, chegaremos a:

(a1 b2 − a2 b1 )y = (a3 b1 − a1 b3 )z.


80

Enfim, temos as seguintes relações:



 (a b − a b )x = (a b − a b )z
1 2 2 1 2 3 3 2
.
 (a b − a b )y = (a b − a b )z
1 2 2 1 3 1 1 3

Para quaisquer valores que consideremos para a variável z, obte-


mos valores para x e y de modo que a terna (x, y, z) é uma solução
do sistema. Isso diz que o sistema possui mais de uma solução. Em
particular, se z = a1 b2 − a2 b1 temos uma solução do sistema, a saber:



 x = a2 b3 − a3 b2

y = a3 b1 − a1 b3 .


 z = a b −a b

1 2 2 1

Chamaremos esta solução de o produto vetorial de u por v, de-


notado por u × v. Ou seja, o produto vetorial de u = (a1 , a2 , a3 ) por
v = (b1 , b2 , b3 ) é

u × v = (a2 b3 − a3 b2 , −(a1 b3 − a3 b1 ), a1 b2 − a2 b1 ).

uxv

Figura 3.7: Produto vetorial

Usando a notação i = (1, 0, 0), j = (0, 1, 0) e k = (0, 0, 1) temos que


(x, y, z) = x · i + y · j + z · k. Agora desenvolvendo o determinante

i j k


a1 a2 a3


b1 b2 b3

81

como se fosse um determinante de terceira ordem sobre o conjunto


dos números reais, obtemos:

i j k


a1 a2 a3 = (a2 b3 − a3 b2 )i − (a1 b3 − a3 b1 )j + (a1 b2 − a2 b1 )k.


b1 b2 b3

Utilizando a definição de produto vetorial e a notação introduzida,


concluı́mos que

u × v = (a2 b3 − a3 b2 , −(a1 b3 − a3 b1 ), a1 b2 − a2 b1 )

= (a2 b3 − a3 b2 )i − (a1 b3 − a3 b1 )j + (a1 b2 − a2 b1 )k.

Portanto,
i j k


u × v = a1 a2 a3 .


b1 b2 b3

Exemplo 3.5.1. Se u = (1, 2, 3) e v = (1, 1, 1), usando o método des-


crito acima, temos:

Para provar a i j k


propriedade 7, 1 2 3 = (2 − 3)i − (1 − 3)j + (1 − 2)k = (−1, 2, −1).


suponha que 1 1 1

u = (a1 , b1 , c1 ) e
Portanto, u × v = (−1, 2, −1).
v = (a2 , b2 , c2 ) e
use as definições As seguintes propriedades do produto vetorial decorrem direta-

de produto vetorial, mente da definição:


produto interno e 1. u × v = −v × u;
norma de um vetor.
2. (u + v) × w = u × w + v × w;

3. (λ · u) × v = u × (λ · v) = λ · (u × v), para todo λ ∈ R;

4. Se u = (a1 , a2 , a3 ), v = (b1 , b2 , b3 ) e w = (c1 , c2 , c3 ) então,



a1 a2 a3


hu × v, wi = det[u, v, w] = b1 b2 b3 .


c1 c2 c3

82

5. O vetor u × v é perpendicular a u e a v;

6. Os vetores u e v são colineares se, somente se, u × v = 0;

7. Vale a seguinte identidade: hu × v, u × vi = |u|2 · |v|2 − hu, vi2.

A última das propriedades apresentadas será utilizada para mos-


trarmos o próximo resultado, por isso, esperamos que o leitor reserve
um pouco de tempo para provar tal propriedade.

Proposição 3.5.1. Quaisquer que sejam os vetores u, v ∈ R3 , não


nulos, tem-se
|u × v| = |u| · |v| sin θ,

onde θ é o ângulo entre u e v.

Demonstração. Observe que,

|u × v|2 = hu × v, u × vi
Uma das mais,
= |u|2 · |v|2 − hu, vi2
senão a mais, im-
= |u|2 · |v|2 − |u|2 · |v|2 cos2 θ
portantes relações
= |u|2 · |v|2 (1 − cos2 θ)
trigonométricas
2 2 2
= |u| · |v| sin θ. (que não pode-
mos esquecer) é a
Da igualdade |u×v|2 = |u|2 ·|v|2 sin2 θ segue que |u×v| = |u|·|v| sin θ
relação fundamen-
Com a propriedade que acabamos de provar, podemos facilmente tal da trigometria:
calcular a área de um triângulo em função de seus vértices. Vejamos.
sin2 θ + cos2 θ = 1,
Sejam A, B e C os vértices de um triângulo e θ a medida do ângulo ∀ θ ∈ R.
 deste triângulo. Então, a origem e os pontos B − A e C − A são
vértices de um triângulo congruente ao triângulo ABC, pois B − A é
−→ −→
um vetor equipolente ao vetor AB e C − A ao vetor AC. Portanto,
esses triângulos têm mesma área.
83

B B−A

A C O C−A

Figura 3.8: Triângulos congruentes

Sendo assim, calcularemos a área do triângulo cujos vértices são


O, B − A e C − A. Desde que a área de um triângulo é o semi-produto
de dois de seus lados e o seno do ângulo formado por esses lados,
vem que

1 1
Area(ABC) = |B − A| · |C − A| sin θ = |(B − A) × (C − A)|.
2 2

Dados três pontos A, B e C, existe um ponto D de modo os pontos


A, B, D e C são, nesta ordem, vértices de um paralelogramo.

B D

A C

Figura 3.9: Paralelogramo ABDC

A área do paralelogramo ABDC é igual a duas vezes a área do


triângulo ABC, então

Area(ABDC) = |(B − A) × (C − A)|.

Exemplo 3.5.2. Calculemos a área do paralelogramo determinado pe-


los vetores (1, 2, 3) e (1, 1, 1). Como os dois vetores têm a origem como
extremiadade comum, segue que a área do paralelogramo vale

i j k


|(1, 2, 3) × (1, 1, 1)| = 1 2 3 = |(−1, 2, −1)| = 6.


1 1 1

84

3.6 Equação do plano

Sejam P um ponto e N um vetor não nulo. Seja π o plano que


passa por P e tal que o vetor N está na direção perpendicular a π.
Entende-se por direção perpendicular a π a direção de qualquer reta
que seja perpendicular a π.

*P

Figura 3.10: Plano perpendicular a N

Portanto, um ponto X ∈ R3 pertence ao plano π se, somente se,


hN, X − P i = 0. A equação hN, X − P i = 0 será chamada de equação
normal de π. O vetor N será chamado de vetor normal a π.
N+P N

P X O X−P

Figura 3.11: Equação do plano

Digamos que X = (x, y, z), P = (p1 , p2 , p3 ) e N = (a, b, c) 6= O,


então a equação do plano que passa em P tendo como vetor normal
o vetor N, em termos das coordenadas de X, P e N, é

hN, X − P i = 0 ⇒ a(x − p1 ) + b(x − p2 ) + c(x − p3 ) = 0

⇒ ax + by + cz = ap1 + bp2 + cp3 .

Consideremos agora a equação geral ax + by + cz = d, em que


a2 + b2 + c2 6= 0. Mostraremos que esta equação representa um plano
que tem como vetor normal o vetor (a, b, c). Suponhamos que a 6= 0.
Os casos b 6= 0 e c 6= 0 são tratados de modo análogo. Então, a
85

equação inicial equivale a a(x − ad ) + b(y − 0) + c(z − 0) = 0. Escrita


assim, reconhecemos que esta equação é a equação de um plano
que passa pelo ponto P = ( ad , 0, 0) tendo como vetor normal o vetor
(a, b, c).

Exemplo 3.6.1. Se P = (1, 1, 1) e N = (1, 2, 3), então a equação do


plano que passa por P tendo N como vetor normal é

x + 2y + 3z = 1 + 2 + 3 = 6.

3.7 Distância de um ponto a um plano

Dado um plano Π e um ponto P 6∈ Π, sabemos que existe uma


única reta r passando por P que é perpendicular a Π. Seja projΠ (P )
o ponto de intersecção da reta r com o plano Π. Chamamos o ponto
projΠ (P ) de projeção ortogonal do ponto P sobre o plano Π.

*P

O ponto projΠ (P ) proj (P)


II

é o ponto do plano
Π mais próximo Figura 3.12: Projeção ortogonal sobre um plano
de P . Podemos
provar este fato
Definição 3.7.1. Dados um plano Π e um ponto P 6∈ Π, definimos a
usando o Teorema
distância de P ao plano Π, denotada por d(P, Π), como
de Pitágoras.
d(P, Π) = d(P, projΠ (P )).

Definição 3.7.2. Dados um plano Π e um ponto P 6∈ Π, definimos o


simétrico de A, em relação ao plano Π, como sendo o ponto A′ tal que
A+A′
projΠ (P ) = 2
.

A seguir, obteremos um fórmula que determina a distância de um


ponto a um plano. Para isso, necessitamos da equação paramétrica
86

de uma reta no espaço R3 . Assim como no plano, se uma reta r passa


pelo ponto P = (p1 , p2 , p3 ) e tem a direção do vetor v = (a, b, c), então
um ponto X = (x, y, z) pertence à reta r se, somente se, o segmento
P X é paralelo ao vetor v. Isto implica que X ∈ π se, somente se,
existe t ∈ R tal que X − P = tv ⇔ existe t ∈ R tal que X = P + tv.

Assim sendo, o conjunto dos pontos X(t) tais que X(t) = P + tv,
em que t ∈ R é a reta que passa no ponto P na direção do vetor v.
Chamaremos t de parâmetro e X(t) = P + tv de equação paramétrica
da reta.

Sejam ax + by + cz = d a equação do plano Π, P = (x0 , y0, z0 ) 6∈ Π e


r a reta que passa por P e é perpendicular a Π. Como o vetor (a, b, c)
é perpendicular a Π, segue que r que tem a mesma direção do vetor
(a, b, c). Assim, a equação paramétrica de r é

X(t) = (x0 + ta, y0 + tb, z0 + tc).

Determinaremos o número real t tal que X(t) = projΠ (P ). Como


X(t) ∈ Π, temos que a(x0 + ta) + b(y0 + tb) + c(z0 + tc) = d. Esta
igualdade implica
d − ax0 − by0 − cz0
t= .
a2 + b2 + c2
Portanto, d(P, Π) = d(P, projΠ (P )) = d(P, X(t)). Consequente-
mente,
q
d(P, Π) = (ta)2 + (tb)2 + (tc)2

= |t| · a2 + b2 + c2
|ax0 + by0 + cz0 − d|
= √ .
a2 + b2 + c2

Em suma, a distância do ponto P = (x0 , y0 , z0 ) ao plano Π, de


equação ax + by + cz = d, é :

|ax0 + by0 + cz0 − d|


d(P, Π) = √ .
a2 + b2 + c2

Podemos reescrever a igualdade encontrada de outra forma. Como


N = (a, b, c) e d = aq1 + bq2 + cq3 , onde Q = (q1 , q2 , q3 ) ∈ Π, temos que
87

a distância do ponto P ao plano Π é

|hN, P − Qi|
d(P, Π) = .
|N|

3.8 Retas reversas

Duas retas são reversas quando têm intersecção vazia e não


são coplanares, ou seja, não estão contidas em um mesmo plano.
Suponha que as equações X1 = P + tu e X1 = Q + tv representam
duas retas reversas r e s, então u e v não são paralelos.
Sejam Π1 o plano que passa por P e tem u × v como vetor normal,
e, Π2 o plano que passa por Q e tem u × v como vetor normal. É claro
que estes planos são paralelos. Perceba também que r ⊂ Π1 , pois o
vetor u × v é perpendicular a u, que está na direção de r, e P ∈ r.
Analogamente, s ⊂ Π2 .

r uxv

Figura 3.13: Retas reversas

De acordo com a notação acima, definimos a distância entre as


retas reversas r e s, denotada por d(r, s), como sendo

d(r, s) = d(Π1 , Π2 ).

Como os planos Π1 e Π2 são paralelos, para obter d(Π1 , Π2 ) basta


calcular a distância de um ponto qualquer de Π1 a Π2 , ou a distância
de um ponto qualquer de Π2 a Π1 .

Observe que as equações dos planos Π1 e Π2 são, respectiva-


mente, hu × v, X − P i = 0 e hu × v, X − Qi = 0. De acordo com o vimos
88

na dedução da fórmula da distância de um ponto a um plano, temos


que
|hu × v, P − Qi|
d(P, Π) = .
|u × v|

3.9 Aplicação

Projeção Estereográfica

Uma projeção tem a propriedade de representar um objeto de n


dimensões em um espaço de dimensão m, em que m ≤ n. A projeção
estereográfica tem a propriedade de levar um objeto de n dimensões
em um espaço de n − 1 dimensões.

A projeção estereográfica consiste em representar a esfera no pla-


no, processo muito utilizado na elaboração de mapas em que represen-
ta-se o globo terrestre em folhas planas.

Figura 3.14: Projeção Estereográfica

Descrição da projeção estereográfica da esfera x2 +y 2 +(z−1)2 = 1


sobre o plano xy: dado um ponto (a, b, c) da esfera, consideremos a
89

semi-reta partindo do polo norte N = (0, 0, 2) e que passa pelo ponto


(a, b, c). Esta semi-reta intersecta o plano xy em único ponto. Este
processo define a projeção estereográfica, que associa a cada ponto
da esfera um ponto do plano.

Dado um ponto P = (a, b, c) pertencente à esfera, vamos determi-


nar o ponto P ′ = (x, y) do plano que está associado a (a, b, c). Veja
que a esfera tem centro no ponto C = (0, 0, 1) e seu raio mede 1.

O ponto (x, y) pode ser visto como um ponto de R3 , neste caso, o


identificamos com o ponto (x, y, 0), isto é, P ′ = (x, y, 0).

A semi-reta partindo do polo norte N = (0, 0, 2) e que passa pelo


−−→
ponto P = (a, b, c) tem a mesma direção do vetor v = NP = (a, b, c−2),
então sua equação paramétria é

R(t) = N + tv = (at, bt, 2 + t(c − 2)), t > 0.

Pela descrição da projeção estereográfica, o ponto P ′ = (x, y, 0) é


dado pela intersecção da semi-reta R(t) com o plano xy. Então, existe
2
t tal que R(t) = (x, y, 0). Donde, 2 + t(c − 2) = 0 ⇒ t = 2−c
. Assim,
2a 2b
x = at = 2−c
e y = bt = 2−c
.

Pelo que foi exposto, concluı́mos que a projeção estereográfica é


Para mais infor- uma aplicação π : S2 {N} → Πxy , onde Πxy é plano xy, dada por:
mações sobre a  2a 2b 
π(a, b, c) = , .
Projeção de Merca- 2−c 2−c
tor, consulte o sitio: Outra projeção bastante utilizada na construção de mapas de nave-
www.mat.uc.pt/ gações é a Projeção de Mercator.
∼picado/geomdif/

3.10 Exercı́cios resolvidos

Q 3.1. Sejam a1 , a2 e a3 números reais positivos, então vale a seguinte


desigualdade:
1 1 1
9≤ + + (a1 + a2 + a3 ).
a1 a2 a3
90
q q q 
1
Solução: Considere os seguintes vetores u = a1
, a12 , a13 e
√ √ √
v = ( a1 , a2 , a3 ), pela desigualdade de Cauchy-Schwarz temos

r 1 r 1 r 1  √ √ √
|hu, vi| ≤ |u| · |v| ⇒ 3 ≤ , , · |( a1 , a2 , a3 )|.

a1 a2 a3

q
1 1 1 √
Donde, 3 ≤ a1
+ a2
+ a3
· a1 + a2 + a3 . Elevando a última desigual-
dade ao quadrado obtemos o resultado desejado.

Q 3.2. Calcule a distância do ponto P = (1, 2, −1) à reta de equação


paramétrica X(t) = (1 + 2t, 5 − t, −2 + 3t).

Solução: Observe que a reta dada tem a mesma direção que o vetor
v = (2, −1, 3). Seja Π o plano que passa por P tendo v como vetor
normal, então sua equação é

2x − y + 3z = 2 · 1 − 1 · 2 − 3 · 1 = −3.

Determinemos a intersecção da reta dada com o plano Π. Seja


t ∈ R tal que X(t) ∈ Π, então

3
2(1 + 2t) − (5 − t) + 3(−2 + 3t) = −3 ⇒ t = .
7

Portanto, a intersecção do plano Π com a reta cuja equação é


X(t) = (1 + 2t, 5 − t, −2 + 3t) é o ponto Q = ( 13
7 7
, 32 , − 57 ). Isso im-

2 91
plica que d(P, X(t)) = d(P, Q) = 7
.

Q 3.3. Obtenha a equação do plano que contém o ponto P e é per-


pendicular ao segmento de reta AB, onde P = (1, 1, −1), A = (3, 5, 2)
e B = (7, 1, 12).

Solução: Seja Π o plano que contém o ponto P e é perpendicular ao


−→
segmento de reta AB, então o vetor v = AB = (4, −4, 10) é normal a
Π. Donde, sua equação é

h(4, −4, 10), (x − 1, y − 1, z + 1)i = 0 ⇒ 4x − 4y + 10z = −10.

Simplificando, temos 2x − 2y + 5z + 5 = 0.
91

Q 3.4. Qual o simétrico do ponto P = (3, 7, 0) em relação ao plano


x + 2y − z = 5?

Solução: Lembremos que o simétrico de P , em relação ao plano


Π : x + 2y − z = 5, é o ponto P ′ tal que projΠ (P ) = 21 (P + P ′). Deste
modo, P ′ = 2 · projΠ (P ) − P . Calculemos projΠ (P ).
Seja r a reta passando por P na direção de (1, 2, −1), então sua
equação paramétrica é X(t) = (3 + t, 7 + 2t, −t). Seja t tal que X(t) ∈
Π, logo
(3 + t) + 2(7 + 2t) − (−t) = 5 ⇒ t = −2.

Segue que projΠ (P ) = X(−2) = (1, 3, 2). Portanto, P ′ = (−1, −1, 4).

Q 3.5. Ache as coordenadas do ponto do plano 2x + y − 2z = 12 que


está mais próximo da origem.

Solução: O ponto do plano Π : 2x+y −2z = 12 que está mais próximo


da origem é projΠ (O), que é dado como a intersecção de Π com a reta
r que passa por O = (0, 0, 0) na direção do vetor (2, 1, −2).
A equação paramétrica de r é X(t) = (2t, t, −2t). Seja t o número
real tal que X(t) ∈ Π, então

4
4t + t + 4t = 12 ⇒ t = .
3

Portanto, o ponto de Π mais próximo da origem é


4 8 4 8
projΠ (0) = X = , ,− .
3 3 3 3

3.11 Exercı́cios propostos

Q 3.6. Dados a1 e a2 números reais positivos. Definimos as médias


Geométrica e Aritmética destes números, respectivamente, por

√ a1 + a2
MG = a1 a2 ; MA = .
2

(a) Mostre que MG ≤ MA;


92

(b) Se a, b e c são números reais positivos, mostre que

8abc ≤ (a + b)(a + c)(b + c).

Q 3.7. Sejam a1 , a2 e a3 números reais positivos. Definimos as médias


Harmônica, Aritmética e Quadrática destes números, respectivamente, Dados a1 , a2 e a3
por números reais posi-

3 a1 + a2 + a3
r
a21 + a22 + a23 tivos, definimos a
MH = 1 1 1 ; MA = ; MQ = .
a1
+ a2
+ a3
3 3 média Geométrica

Prove que MH ≤ MA ≤ MQ. desses números



por MG = 3 a1 a2 a3 .
Q 3.8. Encontre dois vetores perpendiculares, não nulos, ambos para-
Mostra-se que
lelos ao plano x + y + z = 1.
MH ≤ MG ≤ MA.
Q 3.9. Sejam A = (1, 2, 3) e B = (−4, 5, 0). Determine os pontos per-
tencentes ao segmento AB que o dividem em três partes com mesma
medida.

Q 3.10. Determine os valores de a e b para que as retas de equações


X1 (t) = (1 + at, 2 + bt, −1 + 2t) e X2 (t) = (2 + t, 1 + bt, −1 + 2t) sejam:

(a) Paralelas;

(b) Concorrentes;

(c) Reversas.

Q 3.11. Determine os valores de a, b e d para que o plano de equação


ax + by + 3z = d seja:

(a) Paralelo ao plano 2x + y − 5z = 4;

(b) Represente o mesmo plano que 2x + y − 5z = 4.

Q 3.12. Definimos o ângulo entre dois planos como sendo o ângulo


formado por seus vetores normais. Determine o ângulo entre os planos
2x − y + 3z = 0 e x + y − 8z = 1.

Q 3.13. Demonstre que: se u e v são vetores tais que u + v é perpen-


dicular a u − v, então |u| = |v|.
93

Q 3.14. Determine hu, vi + hu, wi + hv, wi, sabendo u + v + w = 0,



|u| = 2, |v| = 3 e |w| = 5.

Q 3.15. Ache um vetor de comprimento 5 simultaneamente perpen-


dicular aos vetores (1, 1, 0) e (2, −1, 3).

Q 3.16. Sabendo que |u×v| = 3 3, |u| = 3 e 60◦ é a medida do ângulo
entre u e v, encontre |v|.

Q 3.17. Calcule a área do paralelogramo gerado pelos vetores (3, 1, 2)


e (4, −1, 0).

Q 3.18. Dados os pontos A = (1, −2, 3), B = (4, 3, −1) e C = (5, 7, −3),
determine o ponto D tal que ABCD, nesta ordem, seja um paralelo-
gramo e calcule sua área.

Q 3.19. Mostre que o ponto (1, −2, 1) não pertence à reta

X(t) = (1 + t, −2t, 1 + t).

Q 3.20. Qual a posição relativa entre a reta X(t) = (1 − t, −1, 2 + 4t) e


o plano 4x − y + z = 7?

Q 3.21. Determine a equação da reta que passa pelos pontos (1, 1, −1)
e (−2, 1, 3).

Q 3.22. Encontre dois planos cuja intersecção é a reta

X(t) = (1 + 3t, 2, 2 + 5t).

Q 3.23. Calcule a distância do ponto (1, 1, 2) ao plano 3x + y − 5z = 2.

Q 3.24. Qual a distância entre o ponto (3, 2, −5) e a reta que passa
nos pontos (1, 0, 1) e (0, 1, −1)?

Q 3.25. Ache a equação da reta que passa no ponto (1, −2, 1) e é


perpendicular à reta X(t) = (1 + t, −2t, 1 + t).

Q 3.26. Sejam A = (1, 0, 1) e Π o plano x + y + z = 1. Ache o ponto


simétrico de A em relação ao plano Π.
94

3.12 Referências Bibliográficas

[1 ] BARSOTTI, L. Geometria analı́tica e vetores. 3a Ed. Local:


Artes Gráficas e Editora Unificado, 1984.

[2 ] BOYER, C. B. História da Matemática. 3a Ed. Local: Editora da


Universidade de São Paulo, 1974.

[3 ] CAMARGO, I. & BOULOS, P. Geometria Analı́tica - um trata-


mento vetorial. 3a Ed. Local: Editora Prentice Hall Brasil, 2005.

[4 ] CAROLI, A. J. de; CALLIOLI, C. A. & FEITOSA, M. O. Vetores,


Geometria Analı́tica: teoria e exercı́cios. 6a Ed. Local: Editora
Nobel, 1968.

[5 ] CAROLI, A.; CALLIOLI, C. & FEITOSA, M. Matrizes Vetores e


Geometria Analı́tica. 17a Ed. Local: Editora Nobel, 1984.

[6 ] IEZZI, G. Fundamentos de Matemática Elementar vol.7: geome-


tria analı́tica. Local: Atual Editora, 2001.

[7 ] LEHMANN, C. H. Geometria Analı́tica. 1a Ed. Local: México -


UTEHA, 1953.

[8 ] LIMA, E. L. Geometria Analı́tica e Álgebra Linear. Local: IMPA


- Coleção Matemática Universitária, 2001.

[9 ] MIDDLEMIS, R. R. Analytic Geometry. 2a Ed. Local: Editora


Mc Graw-Hill Book Company Inc, 1955.

[10 ] MURDOCH, D. C. Geometria Analı́tica: uma introdução ao cál-


culo vetorial e matrizes. 2aEd. Local: Editora Livros Técnicos e
Cientı́ficos, 1971.

[11 ] REIS, G. & SILVA, V. Geometria Analı́tica. 2a Ed. Local: Editora


LTC, 1996.

[12 ] SIMMONS, G. F. Cálculo com Geometria Analı́tica. 1a Ed. Lo-


cal: Editora Mc Graw-Hill, 1987.
95

[13 ] STEINBRUCH, A. & WINTERLE, P. Geometria Analı́tica. 2a Ed.


Local: Editora Makron, 1987.

[14 ] VENTURI, J. Cônicas e Quádricas. 5a Ed. Local: www.geometria


analitica.com.br, 2003.

Web-Bibliografia

[15 ]http://www.geometriaanalitica.com.br/index3.html

[16 ]http://www.paulomarques.com.br/arq6.htm

[17 ]http://pessoal.sercomtel.com.br/matematica/geometria/ganalitica
/ganalitica.htm

[18 ]http://www.ime.unicamp.br/ jardim/livro-GA.pdf

[19 ]http://www.geometriaanalitica.com.br

[20 ]www.mat.uc.pt/ picado/geomdif/


96
RESUMO UNIDADE 4

Introduzimos a definição algébrica das superfı́cies quádricas, fazen-


do uso de equações do segundo grau a três variáveis, e apresenta-
mos as ferramentas necessárias para identificarmos que superfı́cie
quádrica uma equação do segundo grau a três variáveis representa.

Para alcançarmos nosso objetivo, ou seja, identificarmos as quádri-


cas, utilizamos o Teorema Espectral - Diagonalização de Matrizes
Simétricas.
SUMÁRIO

(4.1) Introdução

(4.2) Equações padrão

(4.3) Autovalores e autovetores

(4.4) Conjunto ortonormal

(4.5) Identificação de uma quádrica central

(4.6) Equação geral do segundo grau

(4.7) Nota histórica

(4.8) Exercı́cios resolvidos

(4.9) Exercı́cios propostos

(4.10) Referências Bibliográficas


98
4. Quádricas centrais

4.1 Introdução

Neste capı́tulo, estudaremos as quádricas centrais, que correspon-


dem às superfı́cies, simétricas em relação à origem do sistema de co-
ordenadas, que podem ser representadas por equações do segundo
grau a três variáveis.

Chamamos de equação do segundo grau a três variáveis toda


equação da forma ψ(x, y, z) = J, onde

ψ(x, y, z) = Ax2 + By 2 + Cz 2 + Dxy + Exz + F yz + Gx + Hy + Iz

e A, B, C, D, E, F, G, H, I e J são constantes reais tais que A, B, C, D,


E ou F é diferente de zero. Quando G = H = I = 0 dizemos que a
superfı́cie representada pela equação ψ(x, y, z) = J, ou seja,

Ax2 + By 2 + Cz 2 + Dxy + Exz + F yz = J,

é uma quádrica central. O adjetivo central provém de que

ψ(x, y, z) = ψ(−x, −y, −z),

isto é, o ponto (x, y, z) pertence à superfı́cie se, somente se, o ponto
(−x, −y, −z) também pertence. Logo, a origem O = (0, 0, 0) é um
centro de simetria.

98
99

4.2 Equações padrão

Nesta secção, faremos uma lista de todas as quádricas centrais da


forma
Ax2 + By 2 + Cz 2 = J.

Para isto, necessitamos das três observações abaixo:

• Quando J 6= 0, vale a igualdade Ax2 +By 2 +Cz 2 = J se, somente


se,
A 2 B 2 C 2
x + y + z = 1.
J J J
q
2
Apresentaremos • Se AJ > 0 então, pondo a = J
A
, temos AJ x2 = xa2 . Se AJ < 0
q
2
um exemplo para então, pondo a = − AJ , temos AJ x2 = − xa2 . Analogamente,
2
q q
2
que o leitor entenda temos BJ y 2 = ± yb2 , onde b = ± BJ , e CJ z 2 = ± zc2 , com c = ± CJ .
o que queremos 2 y2 z2 x2 y2 z2
• − xa2 − b2
+ c2
= 1 se, somente se, a2
+ b2
− c2
= −1.
indicar com troca
de nomes dos Destes três fatos resulta que as equações a seguir definem todas
eixos. Exemplo: as quádricas centrais da forma Ax2 + By 2 + Cz 2 = J, exceto por uma
as equações a eventual troca de nomes dos eixos.
seguir representam
x2 y2 z2 x2 y2
a mesma quádrica, 1. a2
+ b2
+ c2
= 1; 7. a2
+ b2
= 1;
x2 y2 z2
a2
+ b2
− c2
= 1 e x2 y2 z2 x2 y2
2. a2
+ b2
+ c2
= −1; 8. a2
− b2
= 1;
x2 y2 z2
a2
− b2
+ c2
= 1,
x2 y2 z2 x2 y2
apenas trocamos 3. a2
+ b2
− c2
= 1; 9. a2
− b2
= 0;

os nomes dos eixos x2 y2 z2 x2


4. a2
+ b2
− c2
= −1; 10. a2
= 1;
y e z.
x2 y2 z2 x2
5. a2
+ b2
+ c2
= 0; 11. a2
= −1;

x2 y2 z2 x2
6. a2
+ b2
− c2
= 0; 12. a2
= 0.

A superfı́cie definda pela equação (1) chama-se Elipsóide. Suas


intersecções com os planos xy, xz e yz são respectivamente as elipses

x2 y 2 x2 z 2 y2 z2
+ = 1 ; + = 1 ; + 2 = 1.
a2 b2 a2 c2 b2 c
100

x2 y2 z2
Figura 4.1: O Elipsóide a2
+ b2
+ c2
=1

Se dois dos três números a, b e c são iguais, tem-se um Elipsóide


2 +z 2
x2
de Revolução. Por exemplo, quando b = c, o Elipsóide a2
+y b2
= 1 é
x2 z2
gerado pela rotação da elipse a2
+ b2
= 1, contida no plano y = 0, em
torno do eixo x.

Figura 4.2: O Elipsóide de Revolução


101

A equação (2) define o conjunto vazio, pois nenhum ponto (x, y, z)


satisfaz tal igualdade.

A superfı́cie definida pela equação (3) é chamada de Hiperbolóide


Para uma melhor
de uma folha. Sua intersecção com o plano xz é a hipérbole
visualização das
x2 z 2
quádricas visite o − 2 = 1.
a2 c
sı́tio:
Por outro lado, a intersecção do Hiperbolóide de um folha com
http://www.mat.
qualquer plano horizontal z = d é a elipse de equação
ufrgs.br/ calculo/
quadrica/ x2 y 2 c2
+ = 1 + .
a2 b2 c2

Figura 4.3: O Hiperbolóide de uma folha

2
z2 2
A equação (4), quando posta sob a forma c2
= 1+ xa2 + yb2 deixa claro
que todos os pontos (x, y, z) que satisfazem esta equação cumprem
a condição |z| ≥ c. Chamamos a superfı́cie definida pela equação (4)
de Hiperbolóide de duas folhas.

A intersecção do Hiperbolóide de duas folhas com qualquer plano


horizontal z = d é a elipse de equação

x2 y 2 d2
+ 2 = −1 + 2 .
a2 b c
102

Figura 4.4: O Hiperbolóide de duas folhas

x2 y2 z2
A equação a2
+ b2
+ c2
= 0, item (5) na lista anterior, é satisfeita
apenas pela origem O = (0, 0, 0).

x2 y2 z2
A equação (6), a2
+ b2
− c2
= 0, representa uma superfı́cie de-
nominada Cone duplo. A intersecção do Cone duplo com um plano
x2 y2 d2
horizontal z = d 6= 0 é a elipse a2
+ b2
= c2
. Perceba que se um
ponto (x, y, z) pertence ao Cone duplo, então para todo número real t
o ponto (tx, ty, tz) também pertence ao Cone duplo.

Figura 4.5: O Cone duplo


103

As soluções da equação (7) são todos os pontos (x, y, z) tais que


x2 y2
a2
+ b2
= 1. Logo, esta equação define o Cilindro reto com base na
x2 y2
elipse a2
+ b2
= 1, isto é, reunião de retas verticais que passam pelos
pontos dessa elipse.

Figura 4.6: O Cilindro Elı́ptico

x2 y2
Analogamente, a superfı́cie definida pela equação (8), a2
− b2
= 1,
x2 y2
é o Cilindro reto com base na hipérbole a2
− b2
= 1, ou seja, reunião
de retas verticais que passam pelos pontos dessa hipérbole.

Figura 4.7: O Cilindro Hiperbólico


104
  
x2 y2 x y x y
A equação (9), a2
− b2
= 0, é equivalente a a
− b a
+ b
= 0.
Ela representa a reunião de dois planos verticais que cortam o plano
x y x y
xy segundo as retas a
− b
=0e a
− b
= 0.

x y x y
Figura 4.8: Planos de equações a
− b
=0e a
+ b
=0

x2
A equação (10), a2
= 1, representa o par de planos verticais cujas
equações são x = ±a.

−a

x
y

Figura 4.9: Planos de equações x = a e x = −a

x2
A equação (11), a2
= −1, representa o conjunto vazio. A equação
x2
(12), a2
= 0, equivalente a x = 0, representa o plano yz.
105

4.3 Autovalores a autovetores


 
m m12 m13
 11 
Dada uma matriz M =  m21 m22 m23  definimos seu polinômio
 
 
m31 m32 m33
caracterı́stico por

m11 − t m12 m13


pM (t) = det(M − tI) = m21 m22 − t m23 .


m31 m32 m33 − t

Exemplo 4.3.1. Calculemos o polinômio caracterı́stico da matriz M


dada por  
0 1 1
 
 1 0 1 .
 
 
1 1 0
Pela definição de pM (t) temos

−t 1 1



pM (t) = 1 −t 1 .


1 1 −t

Efetuando o cálculo do determinante encontra-se a seguinte expressão


pM (t) = −t3 + 3t + 2.

Definição 4.3.1. Os zeros do polinômio pM (t) são chamados de auto-


valores da matriz M.
ANOTE:
................................ Segue diretamente da definição que uma matriz M ∈ M3×3 (R) pos-
................................ sui no máximo três autovalores, pois seu polinômio caracterı́stico tem
................................ grau três.
................................
No exemplo 4.3.1 temos que pM (t) = −t3 + 3t + 2 = −(t − 2)(t + 1)2 ,
................................
então pM (−1) = 0 e pM (2) = 0. Isto diz que os autovalores da matriz
................................
M são −1 e 2. Veja que o autovalor 2 tem multiplicidade 1 e o autovalor
................................
−1 tem ultiplicidade 2.

Observemos que, dizer que o número λ é autovalor da matriz M,


ou seja, que o determinante de M − λI é igual a zero, equivale a
106

afirmar que o sistema linear homogêneo de três equações com três


incógnitas (M−λI)v T = 0 é indeterminado, logo existe alguma solução
v = (x, y, z) não trivial, isto é, v 6= (0, 0, 0). Ora, (M − λI)v T = 0 é o
mesmo que Mv T = λv T .

Portanto, λ é autovalor de M se , somente se, existe algum vetor


v 6= 0 tal que M · v T = λ · v T .
Como já foi dito,
Definição 4.3.2. Se λ ∈ R é autovalor de M, dizemos que um vetor encontrar autove-
v = (x, y, z) é autovetor de M, associado a λ, se tores é equivalente
a encontrar uma
M · vT = λ · vT .
solução particular
de um sistema
Exemplo 4.3.2. Os vetores v1 = (1, 1, 1) e v2 = (1, 0, −1) são autove-
homogêneo de três
tores da matriz  
equações com três
0 1 1
 
 1 0 1 .
  incógnitas.
 
1 1 0
associados a −1 e 2, respectivamente. De fato,
       
0 1 1 1 2 1
       
 1 0 1 · 1 =  2  = 2 ·  1  ⇒ Mv1T = 2v1T .
       
       
1 1 0 1 2 1
       
0 1 1 1 −1 1
       
 1 0 1  ·  0  =  0  = −1 ·  0  ⇒ Mv2T = −1 · v2T .
       
       
1 1 0 −1 1 −1

4.4 Conjunto ortonormal

Um subconjunto X do R3 é dito ortonormal se todos os seus vetores


têm norma 1 e são dois a dois perpendiculares. Segue-se de imediato
que, todo subconjunto ortonormal X possui no máximo três vetores.

Segue da definição que um subconjunto X = {v1 , v2 , v3 } é ortonor-


mal se, só se, |v1 | = |v2 | = |v3 | = 1 e hv1 , v2 i = hv1 , v3 i = hv2 , v3 i = 0.
107

Exemplo 4.4.1. O conjunto X = {( √13 , √13 , √13 ), ( √12 , 0, − √12 )} é ortonor-


mal. De fato,
D   E
• √1 , √1 , √1 , √1 , 0, − √1 = √1 − √1 = 0;
3 3 3 2 2 6 6
 q
• √13 , √13 , √13 ) = 13 + 13 + 31 = 1;

  q
• √12 , 0, − √12 = 12 + 12 = 1.

A fim de obtermos ferramentas para classificarmos as quádricas


centrais necessitamos do teorema a seguir, o mesmo garante que
toda matriz simétrica é semelhante a uma matriz diagonal.

Teorema 4.4.1 (Diagonalização de Matrizes Simétricas). Dada uma


O conjunto ortonor- matriz simétrica
 
mal X, dado no m11 m12 m13
 
M =  m21 m22 m23
 
teorema, é formado 
 
por autovetores m31 m32 m33
da matriz M, existe um conjunto ortonormal X = {(a1 , a2 , a3 ), (b1 , b2 , b3 ), (c1 , c2 , c3 )}
então, estamos tal que
afirmando que toda
     
matriz simétrica m11 m12 m13 a1 a2 a3 λ1 0 0 a1 b1 c1
     
M ∈ M3×3 (R) pos-  m21 m22 m23  =  b1 b2 b3  ·  0 λ2 0 · a b c ,
     
 2 2 2 
 
   
sui três autovetores m31 m32 m33 c1 c2 c3 0 0 λ3 a3 b3 c3
ortonormais. onde as constantes λ1 ≤ λ2 ≤ λ3 são os autovalores de M.

O teorema acima, conhecido como Teorema Espectral, garante


que toda matriz simétrica M ∈ M3×3 (R) possui três autovalores, mais
não garante que os mesmos são distintos.

A matriz P , dada no teorema 4.4.1, cujas linhas são os vetores


(a1 , a2 , a3 ), (b1 , b2 , b3 ) e (c1 , c2 , c3 ) satisfaz a seguinte igualdade matri-
cial: P P T = P T P = I3 (desafiamos o leitor a verificar a veracidade
desta igualdade). As matrizes que satisfazem a esta igualdade são
chamadas de matrizes ortogonais.

A igualdade P P T = P T P = I3 diz que a matriz P é invertı́vel e que


P −1 = P T .
108

Definição 4.4.2. Dizemos que uma matriz ortogonal P é uma mudan-


ça de coordenadas cartesianas.

A definição anterior tem o seguinte sentido: sejam e1 = (1, 0, 0),


e2 = (0, 1, 0), e3 = (0, 0, 1) e P uma matriz ortogonal, então os vetores
v1 , v2 e v3 , dados por v1T = P eT1 , v2T = P eT2 e v3T = P eT3 , formam um sis-
tema de coordenadas cartesianas. Por sistema de coordenadas carte-
sianas entende-se três direções unitárias e perpendiculares entre-se.

Usando as letras r, s e t para denotarmos as coordenadas relativas


aos eixos v1 , v2 e v3 , respectivamente, vale a seguinte relação:
     
r a b c x
   1 1 1   
 s  =  a2 b2 c2  ·  y  .
     
     
t a3 b3 c3 z

t
z

s
y

x r

Figura 4.10: Mudança de coordenadas

4.5 Identificação de uma quádrica central

Nesta secção, veremos um processo algébrico para transformamos


uma equação geral, que representa uma quádrica central, em uma
equação a três variáveis que se exprime como combinação linear dos
quadrados das coordenadas.

Lembremos que uma quádrica central é representada por uma


equação do tipo

Ax2 + By 2 + Cz 2 + Dxy + Exz + F yz = J.


109

Podemos reescrever esta igualdade na forma matricial que segue


abaixo:    
D E
A 2 2
x
h i    
D F
x y z · B  ·  y  = J.
   
 2 2
  
E F
2 2
C z
 
D E
A 2 2
 
Chamaremos a matriz  D F  de matriz associada à quádrica
B
 
 2 2 
E F
C 2 2
2 2 2
central Ax + By + Cz + Dxy + Exz + F yz = J.

Pelo Teorema Espectral, segue que


       
A D2 E2 a1 a2 a3 λ 0 0 a1 b1 c1
     1   
 D
 2 B F2  =  b1 b2 b3  ·  0 λ2 0  ·  a2 b2 c2  ,
      
       
E F
2 2
C c1 c2 c3 0 0 λ3 a3 b3 c3

onde as constantes λ1 ≤ λ2 ≤ λ3 são os autovalores da matriz asso-


ciada à quádrica central Ax2 + By 2 + Cz 2 + Dxy + Exz + F yz = J.

Substituindo a igualdade matricial encontrada na equação das quá-


dricas centrais, obtemos
    
a a a λ1 0 0 a1 b1 c1 x
h i 1 2 3    
dadas x y z · b b b ·  0 λ2 0 · a b c · y  = J.
    
Lembrete:  1 2 3   2 2 2  
 
duas matrizes (A e c1 c2 c3 0 0 λ3 a3 b3 c3 z
B), de modo que  
a a a
exista AB, vale a h i h i 1 2 3 
Agora observe que r s t = x y z · b1 b2 b3  , então
 
seguinte relação  
c1 c2 c3
(AB)T = B T AT ,  
λ 0 0
onde M T denota a h i  1  h i
r s t ·  0 λ2 0  · r s t = J.
 
transposta de M.  
0 0 λ3
Portanto, a quádrica central em análise, considerada no novo sis-
tema de coordenadas cartesianas - v1 = (a1 , a2 , a3 ), v2 = (b1 , b2 , b3 ) e
v3 = (c1 , c2 , c3 ) - é representada pela equação: λ1 r 2 +λ2 s2 +λ3 t2 = J. A
equação encontrada pode ser comparada com as equações padrão,
110

que representam as quádricas centrais, e com isto identificarmos qual


a quádrica central em questão.

Exemplo 4.5.1. Classificar  central xy + xz


 a quádrica + yz = 1. A
0 12 21 −t 21 1

2

 
matriz associada é M =  2 0 21  , então pM (t) = 21 −t 12 .
 1 
 
1 1 1 1
0 −t

2 2 2 2

ANOTE:
Donde, pM (t) = −t3 + 34 t + 14 . Observe que pM (t) = −(t − 1)(t + 21 )2 .
................................
Isto diz que 1 é autovalor de M com multipliciade 1 e − 21 é autovalor
................................
de M com multipliciade 2. Portanto, nos novos eixos coodenados, a
................................
quádrica se exprime como
................................
1 1 ................................
− r 2 − s2 + t2 = 1.
2 2 ................................
Comparando com as equações padrão, concluı́mos que a equação ................................
xy + xz + yz = 1 representa um hiperbolóide de duas folhas.

4.6 Equação geral do segundo grau

Uma quádrica, não necessariamente central, é uma superfı́cie cuja


representação analı́tica é uma equação do segundo grau a três variá-
veis, isto é, uma equação da forma ψ(x, y, z) = J, onde

ψ(x, y, z) = Ax2 + By 2 + Cz 2 + Dxy + Exz + F yz + Gx + Hy + Iz.

Mostra-se que tais superfı́cies são as quádricas centrais, que exami-


namos anteriormente, mais os parabolóides. As equações dos parabo-
lóides são basicamente duas, a saber:

• z = Ax2 + By 2 + C, com A > 0 e B > 0;

• z = Ax2 − By 2 + C, com A > 0 e B > 0.

Foge ao objetivo analisar os parabolóides, mas apresentaremos a


seguir as figuras correspondentes aos dois casos citados.
111

Figura 4.11: O Parabolóide Elı́ptico

Figura 4.12: O Parabolóide Hiperbólico


112

4.7 Nota histórica

Resenha histórica

Fulcrado nos geômetras gregos e no desenvolvimento da Álgebra


em toda Europa, Pierre de Fermat conclui em 1629 o manuscrito
Ad locos planos et solidos isagoge (Introdução aos lugares planos
e sólidos). Embora haja controvérsias, tal manuscrito representa o
marco zero da Geometria Analı́tica. É evidente em Fermat a percepção
de uma Geometria Analı́tica em três dimensões: ”Se o problema pro-
posto envolve três incógnitas, deve-se achar, para satisfazer a equação,
não apenas um ponto ou uma curva, mas sim uma superfı́cie”.

A partir de Fermat, a Geometria Analı́tica trouxe inúmeras facili-


dades ao desenvolvimento da Geometria Plana e Espacial e foi con-
siderada a ”estrada real”, numa alusão a Euclides que afirmava ao
rei Ptolomeu que ”não havia nenhuma estrada real para se aprender
geometria”.

Muitas das obras de Euclides (século III a.C.) se perderam. Mas


há consistentes referências que o grande geômetra tinha escrito um
tratado sobre elipsóides, parabolóides, hiperbolóides, além da esfera,
cilindro e cone.

Euclides fundou a Escola de Matemática na renomada Biblioteca


de Alexandria, que pode ter alcançado a cifra de 700.000 rolos (papiros
e pegaminhos).

A Biblioteca de Alexandria está muito próxima do que se entende


hoje por Universidade. Alexandria, a partir de Euclides, até o século
IV a.C., reinou quase absoluta não só como a mais eclética e cos-
mopolita cidade da Antiguidade, mas também como principal centro
da produção matemática.

Em 640 d.C., o califa Omar mandou que fossem queimados todos


os livros da Biblioteca sob o argumento de que ”ou os livros contêm o
113

que está no Alcorão e são desnecessários ou contêm o oposto e não


devemos lê-los”.

Por sua vez, Arquimedes (287-212 a.C.) legou-nos uma original


e vastı́ssima produção em Geometria Plana e Sólida. Arquimedes
nasceu e foi morto em Siracura, na ilha grega de Secı́lia. Quando
jovem, estudou em Alexandria, com os discı́pulos de Euclides. A
genialidade de Arquimedes como fı́sico-matemático só é comparável
com Isaac Newton.

Há dois tratados de Arquimedes que apresentam uma extraordiná-


ria profundidade em relação aos sólidos de revolução. Sobre conói-
des e esferóides: descreve sólidos de revolução gerados por elipses,
parábolas e hipérboles em torno dos seus eixos (quádricas de revolu-
ção). Ademais, neste tratado, Arquimedes obtém a área de uma elipse
(A = abπ). Sobre esfera e cilindro: contém demonstrações rigorosas
do cálculo do volume e da área dos referidos sólidos. Vai além: es-
tuda as áreas e volumes das superfı́cies obtidas por secções planas
sobre a esfera (calotas e segmentos) e sobre o cilindro. A pedido de
Arquimedes, foi gravada na lápide de seu túmulo a representação de
uma esfera inscrita num cilindro circular reto.

Do que atualmente denomina-se quádricas, há contribuições sig-


nificativas de dois outros matemáticos: Apolônio e Pappus. Apolônio
de Perga (c. 262-190 a.C.), que se supõe tenha estudado e por al-
gum tempo ensinado na ”Universidade” de Alexandria, mostrou pela
primeira vez que a elipse, a parábola e a hipérbole podem ser obti-
das variando a inclinação do plano de secção sobre um cone de duas
folhas. Para gáudio de todos, sua monumental obra As Cônicas so-
breviveu praticamente incólume: dos oito livros apenas um se perdeu.
Pappus (século IVd.C.) viveu quando a matemática grega dava seus
últimos suspiros. Escreveu Coleção Matemática, que infelizmente
foi um ”requiem da matemática grega, porque depois de Pappus, a
matemática estagnou e quase desapareceu, e teve de esperar por
114

1.300 anos para um renascimento no começo do século XVII” (George


F. Simmons).

A Coleção Matemática é uma espécie de enciclopédia onde se faz


um epı́tome com os devidos créditos das descobertas dos matemáti-
cos gregos. Por ser bastante criterioso e escrupuloso na citação das
fontes, credita-se a Pappus belos teoremas da Geometria sobre Cen-
tros de Gravidade de Sólidos e Superfı́cies de Revolução.

Entretanto, deve-se a Leonhard Euler (1707-1783) uma das mais


significativas contribuições á geometria no R3 . Em seu livro Intro-
ductio in Analysin Infinitorum (Introdução à Análise Infinita, pub-
licado em 1748) apresenta a primeira exposição em livro-texto de
quádricas, considerando estas como superfı́cies do 2◦ grau no R3 .
No mencionado livro, Euler apresenta as equações dos cones, dos
parabolóides, dos elipsóides e dos hiperbolóides, utilizando sistema
de coordenadas no R3 .

Euler, suı́ço de nascimento, escrevia em média 800 páginas por


ano e a coletânea completa de suas obras é composta de cerca de
75 volumes. Em plena atividade intelectual, morreu aos 76 anos,
sendo que os últimos17 passou em total segueira (consequência de
uma catarata). Mesmo cego, continuou ditando suas descobertas
matemáticas aos 13 filhos.

Euler se ocupou praticamente com todos os ramos então conheci-


dos da Matemática, a ponto de merecer do françês François Arago
o seguinte encômio: ”Euler calculava sem qualquer esfoço aparente,
como os homens respiram e as águias se sustentam no ar”.

A partir do século XVII, superfı́cies têm um notável incremento


com o surgimento da Geometria Diferencial com interaplicações do
Cálculo Diferencial e Integral e da Geometria Analı́tica.
115

4.8 Exercı́cios resolvidos

Q 4.1. Seja ϕ(x, y, z) = x2 + 4y 2 + z 2 + 4xy + 2xz + 4yz, identifique que


quádrica a equação ϕ(x, y, z) = d representa.
 
1 2 1
 
Solução: A matriz associada à quádrica é M =  2 4 2  , então
 
 
1 2 1

1−t 2 1


pM (t) = 2 4−t 2 . Efetuando o cálculo do determinante


1 2 1−t

encontramos pM (t) = −t2 (t − 2). Portanto, nos novos eixos coordena-


dos, a quádrica se exprime como

2t2 = d.

Se d < 0, então 2t2 = d não tem solução, ou seja, representa o


conjunto vazio. Quando d = 0, define o plano t = 0 e, quando d > 0, a
q
equação representa o par de planos paralelos de equações t = ± d2 .

4.9 Exercı́cios propostos

Q 4.2. Em cada caso abaixo, identifique que quádrica a equação


ϕ(x, y, z) = d representa.

(a) ϕ(x, y, z) = x2 + 2y 2 + 3z 2 − xy − 2xz − 3yz

(b) ϕ(x, y, z) = x2 + 5y 2 + z 2 − 4xy + 2xz − 4yz

(b) ϕ(x, y, z) = x2 + 3y 2 + z 2 + 4xy + 2xz + 4yz

(d) ϕ(x, y, z) = 4x2 + 3y 2 − z 2 − 12xy + 4xz − 8yz

(e) ϕ(x, y, z) = −x2 − y 2 − 7z 2 + 16xy + 8xz + 8yz

(f) ϕ(x, y, z) = 2x2 + y 2 − 4xy − 4yz

(g) ϕ(x, y, z) = 3x2 + y 2 − 2xy − 6xz − 6yz


116

(h) ϕ(x, y, z) = −2x2 + 4y 2 + 6z 2 + 2xy + 6xz + 6yz

(i) ϕ(x, y, z) = 4x2 + y 2 − 8z 2 + 4xy − 4xz + 8yz

(j) ϕ(x, y, z) = 3x2 + 3z 2 + 4xy + 8xz + 4yz


117

4.10 Referências Bibliográficas

[1 ] BARSOTTI, L. Geometria analı́tica e vetores. 3a Ed. Local:


Artes Gráficas e Editora Unificado, 1984.

[2 ] BOYER, C. B. História da Matemática. 3a Ed. Local: Editora da


Universidade de São Paulo, 1974.

[3 ] CAMARGO, I. & BOULOS, P. Geometria Analı́tica - um trata-


mento vetorial. 3a Ed. Local: Editora Prentice Hall Brasil, 2005.

[4 ] CAROLI, A. J. de; CALLIOLI, C. A. & FEITOSA, M. O. Vetores,


Geometria Analı́tica: teoria e exercı́cios. 6a Ed. Local: Editora
Nobel, 1968.

[5 ] CAROLI, A.; CALLIOLI, C. & FEITOSA, M. Matrizes Vetores e


Geometria Analı́tica. 17a Ed. Local: Editora Nobel, 1984.

[6 ] IEZZI, G. Fundamentos de Matemática Elementar vol.7: geome-


tria analı́tica. Local: Atual Editora, 2001.

[7 ] LEHMANN, C. H. Geometria Analı́tica. 1a Ed. Local: México -


UTEHA, 1953.

[8 ] LIMA, E. L. Geometria Analı́tica e Álgebra Linear. Local: IMPA


- Coleção Matemática Universitária, 2001.

[9 ] MIDDLEMIS, R. R. Analytic Geometry. 2a Ed. Local: Editora


Mc Graw-Hill Book Company Inc, 1955.

[10 ] MURDOCH, D. C. Geometria Analı́tica: uma introdução ao cál-


culo vetorial e matrizes. 2aEd. Local: Editora Livros Técnicos e
Cientı́ficos, 1971.

[11 ] REIS, G. & SILVA, V. Geometria Analı́tica. 2a Ed. Local: Editora


LTC, 1996.

[12 ] SIMMONS, G. F. Cálculo com Geometria Analı́tica. 1a Ed. Lo-


cal: Editora Mc Graw-Hill, 1987.
118

[13 ] STEINBRUCH, A. & WINTERLE, P. Geometria Analı́tica. 2a Ed.


Local: Editora Makron, 1987.

[14 ] VENTURI, J. Cônicas e Quádricas. 5a Ed. Local: www.geometria


analitica.com.br, 2003.

Web-Bibliografia

[15 ]http://www.geometriaanalitica.com.br/index3.html

[16 ]http://www.paulomarques.com.br/arq6.htm

[17 ]http://pessoal.sercomtel.com.br/matematica/geometria/ganalitica
/ganalitica.htm

[18 ]http://www.ime.unicamp.br/ jardim/livro-GA.pdf

[19 ]http://www.geometriaanalitica.com.br

[20 ]www.mat.uc.pt/ picado/geomdif/

Você também pode gostar